Thursday, September 15, 2011

Some varieties of atheism

A religion typically has both practical and theoretical aspects.  The former concern its moral teachings and rituals, the latter its metaphysical commitments and the way in which its practical teachings are systematically articulated.  An atheist will naturally reject not only the theoretical aspects, but also the practical ones, at least to the extent that they presuppose the theoretical aspects.  But different atheists will take different attitudes to each of the two aspects, ranging from respectful or even regretful disagreement to extreme hostility.  And distinguishing these various possible attitudes can help us to understand how the New Atheism differs from earlier varieties.

Consider first the different attitudes an atheist might take to the theoretical side of a religion.  There are at least three such attitudes, which, going from the most hostile to the least hostile, could be summarized as follows:

1. Religious belief has no serious intellectual content at all.  It is and always has been little more than superstition, the arguments offered in its defense have always been feeble rationalizations, and its claims are easily refuted.

2. Religious belief does have serious intellectual content, has been developed in interesting and sophisticated ways by philosophers and theologians, and was defensible given the scientific and philosophical knowledge available to previous generations.  But advances in science and philosophy have now more or less decisively refuted it.  Though we can respect the intelligence of an Aquinas or a Maimonides, we can no longer take their views seriously as live options.

3. Religious belief is still intellectually defensible today, but not as defensible as atheism.  An intelligent and well-informed person could be persuaded by the arguments presented by the most sophisticated contemporary proponents of a religion, but the arguments of atheists are at the end of the day more plausible.

Obviously one could take one of these attitudes towards some religions, and another of them towards other religions.  For example, a given atheist might take a type 1 atheist position with respect to Christianity and a type 2 atheist position with respect to Buddhism (or whatever).  Or he might take a type 1 attitude towards some versions of Christianity but a type 2 or type 3 attitude towards other versions of Christianity.

Now, among well-known atheists, it seems to me that Quentin Smith is plausibly to be regarded as taking a type 3 attitude toward Christianity, at least as Christianity is represented by prominent philosophers of religion like William Lane Craig or Alvin Plantinga.  Keith Parsons, by contrast, seems to take at best a type 2 attitude towards Christianity and maybe even a type 1 attitude.  And Jerry Coyne seems almost certainly to take a type 1 attitude, though perhaps on a good day and with respect to at least some varieties of religious belief he’d move up to type 2.  (I’m happy to be corrected by Smith, Parsons, or Coyne if I’ve got any of them pegged wrong.)

Now let’s consider three different attitudes an atheist could take toward the practical side of a religion, going again from the most hostile to the least hostile:

A. Religious practice is mostly or entirely contemptible and something we would all be well rid of.  The ritual side of religion is just crude and pointless superstition.  Religious morality, where it differs from secular morality, is sheer bigotry.  Even where certain moral principles associated with a particular religion have value, their association with the religion is merely an accident of history.  Moreover, such principles tend to be distorted by the religious context.  They certainly do not in any way depend on religion for their justification.

B. Religious practice has a certain admirable gravitas and it is possible that its ritual and moral aspects fulfill a real human need for some people.  We can treat it respectfully, the way an anthropologist might treat the practices of a culture he is studying.  But it does not fulfill any universal human need, and the most intelligent, well educated, and morally sophisticated human beings certainly have no need for it.  

C. Religious practice fulfills a truly universal or nearly universal human need, but unfortunately it has no rational foundation and its metaphysical presuppositions are probably false.  This is a tragedy, for the loss of religious belief will make human life shallower and in other ways leave a gaping void in our lives which cannot plausibly be filled by anything else.  It may even have grave social consequences.  But it is something we must find a way to live with, for atheism is intellectually unavoidable.

Here too a given atheist might of course take attitude A towards some religions or some forms of a particular religion, while taking attitude B or C towards others.  Once again, Jerry Coyne seems to be an example of an atheist whose attitude toward religion lays more or less at the most negative end (A).  Perhaps Stephen Jay Gould took something like attitude B.  Atheists of a politically or morally conservative bent typically take either attitude B or attitude C (though I know at least one prominent conservative who is probably closer to attitude A).  Walter Kaufmann is another good example of an atheist (or at least an agnostic) who took something like attitude B towards at least some forms of religion.  Indeed, he seemed to regard religion as something that speaks to deep human needs and whose moral aspects are of great and abiding philosophical interest.

Now these two sets of possible attitudes can obviously be mixed in a number of ways.  That is to say, a given atheist might take a more negative attitude towards the theoretical side of a given religion and a more positive attitude towards its practical side, or vice versa.  And he might take different mixtures of attitudes towards different religions or forms of religion.  For instance, he might take attitudes 2 and C towards some kinds of religious belief, and 1 and A towards other kinds.  Thus we could classify atheists according to their combinations of attitudes towards the practical and theoretical sides of religion or of a particular religion -- A1, B3, C2, and so forth.  

An A1 atheist, then, would be the most negative sort, especially if he took an A1 attitude towards most or all forms of religion.  A C3 atheist would be the most positive.  At different times during my own years as an atheist, I would say that I tended to take either a B or C attitude towards the practical side of religion, and perhaps attitude 2 towards the theoretical side (at least until the latter part of my atheist years, when I started to move to 3 before finally giving up atheism).  No doubt I had moments when I probably came across as more of an attitude 1 and/or attitude A type atheist with respect to at least some forms of religious belief -- it’s easier to remember specific arguments with people than what one’s general attitude was during a given year, say -- but overall I’d say that I probably hovered around B2 territory for at least much of my time as an atheist.  (Walter Kaufmann was one of my heroes in those days.  Indeed, Kaufmann’s attitude towards Christianity -- which was more negative than his attitude towards other religions -- influenced my own, and no doubt helped delay my eventual return to the Church.)

I find that atheists who fall on the most negative ends of these scales -- A1 territory -- are invariably the ones who are the least well-informed about what the religions they criticize actually believe, and the least rational when one tries to discuss the subject with them.  And when you think about it, even before one gets into the specifics it is pretty clear that A1 is prima facie simply not a very reasonable attitude to take about at least the great world religions.  To think that it is reasonable, you have to think it plausible that the greatest minds of entire civilizations -- Augustine, Aquinas, Maimonides, Avicenna, Averroes, Lao Tzu, Confucius, Mencius, Buddha, Adi Shankara, Ramanuja, et al. -- had for millennia been defending theoretical and practical positions that were not merely mistaken but were in fact nothing more than sheer bigotry and superstition, more or less rationally groundless and morally out of sync with the deepest human needs.  And that simply isn’t plausible.  Indeed, it’s pretty obviously ridiculous.  Even if all religious belief turned out to be wrong, it simply is not at all likely that its key aspects -- and especially those aspects that recur in most or all religions -- could have survived for so long across so many cultures and attracted the respect of so many intelligent minds unless they had some significant appeal both to our intellectual and moral natures.  Hence a reasonable atheist should acknowledge that it is likely that attitudes 2 or 3 and B or C are the more defensible attitudes to take towards at least the ideas of the greatest religious thinkers and the most highly developed systems of religious thought and practice. 

When one considers the prima facie implausibility of the A1 attitude together with the ill-informed smugness and irrationality of those who approximate it, it is pretty clear that its roots are not intellectual but emotional -- that it affords those beholden to it a sense of superiority over others, an enemy on which to direct their hatreds and resentments, a way to rationalize their rejection of certain moral restraints they dislike, and so forth.  In other words, A1 atheism is pretty much exactly the sort of ill-informed bigotry and wish-fulfillment A1 atheists like to attribute to religious believers.

And here’s the thing: If there is anything new about the New Atheism, it is the greater prominence of atheists who at least approximate the A1 stripe.  In Walter Kaufmann’s day, A1 atheism was represented by marginal, vulgar cranks like Madalyn Murray O’Hair.   Now, equally vulgar cranks like Dawkins, Harris, Hitchens, Myers, and Coyne are by no means marginal, but widely regarded as Serious Thinkers.  This is the reverse of intellectual progress.  And we know what Walter Kaufmann would have thought of it.

741 comments:

  1. Now, equally vulgar cranks like Dawkins, Harris, Hitchens, Myers, and Coyne are by no means marginal, but widely regarded as Serious Thinkers.

    What to do? Insult them or argue for spirituality rather than take it as a given. You no longer have the Jesuit option to the extent that it use to exist. Allowing people to think for themselves is a risky business.

    ReplyDelete
  2. ...its claims are easily refuted.

    I doubt you will find any atheist who can refute claims that are not empirically testable so unless you reword it (for example factual claims) you won't get many takers for option 1. Dawkins doesn't rule out God's existence, he merely thinks it very unlikely.

    ReplyDelete
  3. Your A, B and C options are unsatisfactory. If you listed all the individual points and asked for a response from a representative group of atheists (oxymoron though I suspect that is) from "strongly disagree" to "strongly agree" I doubt there would be much correlation to your three categories.

    It's testable too. We could count horses teeth!

    ReplyDelete
  4. Alan Fox,

    What to do? Insult them or argue for spirituality rather than take it as a given.


    Ridicule them to the fullest. Since they are truly a joke.


    You no longer have the Jesuit option to the extent that it use to exist.

    LOL LOL LOL LOL LOL!!


    This proves what I wrote earlier


    Allowing people to think for themselves is a risky business.

    Yes. Just look what happened to you.

    ReplyDelete
  5. Ridicule them to the fullest.

    That's OK if it stops there. Definitely no pitchforks! I don't think we should seek to persuade others of our belief or lack of it unless they at least express a passing interest. Live and let live, as the Catholic Churh as always done. ;)

    ReplyDelete
  6. Edward,

    Putting aside the fact that you apparently hit a nerve with Fox (Wow Alan, 5 replies right away, and with terribly little content other than nervous slogans. Work those issues out!)...

    I suppose your 2 lists aren't exhaustive, are they? In particular it seems that, given 3, something less hostile than C must obtain. (Since 3 suggests that religious belief can, in fact, have a rational basis, but C commits to otherwise.)

    I do agree about the vulgar cranks, though.

    ReplyDelete
  7. What to do? Insult them or argue for spirituality rather than take it as a given.

    Yeah, if only Ed had written a whole book doing that before spouting off, right!?

    You no longer have the Jesuit option to the extent that it use to exist.

    Ed, you should've mentioned that A1 atheists tend to be historically ignorant too.

    ReplyDelete
  8. If only we had more of the B2-C3 types of atheist hanging around the comboxes we might get fruitful discussion, rather than troll-like nonsense. It seems most of your critics fall into the A1 type of atheist, and they don't seem to be able to get past their own opinions and actually read what you say. They seem content listening the their A1 followers echoing one another in their own comboxes and being satisfied that they have heard 'all' the arguments.

    Lets have more Niezsches people and less Dawkins.

    ReplyDelete
  9. Alan Fox

    If you find Fundamentalist A1 Atheism so attractive that is your choice.

    But nobody here will take you seriously. Just look at how djindra is treated. He has nothing going on in between his ears. Just a lot of sound an fury signifying nothing.

    >Dawkins doesn't rule out God's existence, he merely thinks it very unlikely.

    Dawkins views "god" as an entity more complex than the universe and postulates given such a psychotically high degree of complexity It's existence is very very (add ten thousand very's) improbable.

    I (and this will shock you) agree with him. So Dawkins (who is incompetent in the area of philosophy) shows us an entity resembling Azathoth from HP Lovecraft probably does not exist.

    Good I can sleep at night. But YHWH historically is viewed to have no parts or passions. His substance is simple. He contains no potencies or composites so how can Dawkins apply his Boeing argument to such an entity.

    Or to put is more simply for an A1 Atheist to comprehend. How does refuting the existence of a "god" I don't believe exists in the first place get you anywhere?

    It simply doesn't.

    Cheers.

    ReplyDelete
  10. I get a kick out of those people, say in the Post Office line, who insult those they deal with and go through life angry that everyone around them is an idiot. The thing is, people behave towards you like you project to them. That's the problem with a lot of religionists today. They ignore the fact that their own behavior is the source of the problem. Look in the mirror, Ed.

    ReplyDelete
  11. This is really interesting, but these classifications may only be useful up to a point. Atheist attitudes vary not just with regard to different religions, but also with regard to different historical eras – because the atheist narrative leans heavily on the idea of gradual scientific enlightenment.

    Belligerent comboxers aside, I suspect quite a lot of people are vaguely B2 or A2 towards ‘the greatest minds of entire civilizations’, because Augustine and so on had the excuse of scientific ignorance, but feel free to be A1 towards present-day religion, because present-day believers supposedly have no excuse.

    Of course, atheists can only be ambivalent in this way if they are ignorant of what Augustine, etc., believed, and of the continuity between ancient and modern religious belief. But then, that is exactly what most atheists are ignorant of.

    The classification is also difficult to apply to past atheists like Bayle and Hume, because they were concealing their beliefs so it is hard to work out what they thought.

    Between the two, however – say, for the atheist thinkers of the nineteenth and twentieth centuries – the system is very handy.

    William Peaden, I agree about the need for ‘more Nietzsches’, but he would need his own separate classification!

    ReplyDelete
  12. @ djindra

    I think it's not Ed who's projecting here

    The thing is, people behave towards you like you project to them.

    That is not always true. Besides Edward was making a scope of different types of view (amd yours is clearly A1)

    That's the problem with a lot of religionists today.

    Because atheists like PZ Meyer are all roses and sunshine.

    They ignore the fact that their own behavior is the source of the problem. Look in the mirror, Ed.

    I think YOU are projecting here

    Logically most people who reply to you in blogs comboxes must tell you you are an idiot or most probably just ignore you when they find out you are just a troll.

    Indeed I have noticed that even here no one even bothers to reply to your nonsense, and you have not been here long (shame o me for trying I guess)

    So I think YOU are the one projectin here... you are projecting your frustrations as a 2 cents atheist so hard you could be a movie projector.

    YOU should get a very good look in the metaphorical mirror and rething your life and your behavior.

    ReplyDelete
  13. What to do? Insult them or argue for spirituality rather than take it as a given. You no longer have the Jesuit option to the extent that it use to exist. Allowing people to think for themselves is a risky business.

    Dawkins, Myers & Co. ARE vulgar. They have an utterly vulgar behavior... vulgar and utterly stupid.

    Read PZ Myer's blog sometimes... funny it is hosted in 'scienceblogs' but it is mostly a series of incoherent rants. It's shameful. The same could be said for Dawkins' site/forum.

    If that's the pinnacle of atheist reason I do not want to see the bottom!

    Dawkins doesn't rule out God's existence, he merely thinks it very unlikely.


    More than once Dawkins has claimed that believing in God is like believing in the tooth fairy… if he claimed that God is merely ‘improbable’ to him, it was maybe on one of his best days whn the bile level was low.

    His book is called the God DELUSION, not the God 'Improbability'.

    Besides Dawkins & Posse do not need you to defend them, they are vicious enough as they are.

    ====================

    People, Prof. Feser wrote a blog post, not a 100 pages study on the moieties of atheism.

    I think he represented the general groups quite well, but of course there are bound to be more subtleties than the 9 possible combinations from A1 to C3 (although someone like A3 or a C1 must be quite rare I suppose).

    ReplyDelete
  14. Dan,

    Of course, atheists can only be ambivalent in this way if they are ignorant of what Augustine, etc., believed, and of the continuity between ancient and modern religious belief. But then, that is exactly what most atheists are ignorant of.

    That, and a careful distinction between natural science and philosophy. Most of the ones around here end up being some form of positivist.

    ReplyDelete
  15. From my reading of Coyne, he is most accurately categorized as A-B2, not A1.

    JM

    ReplyDelete
  16. Alan Fox? Same Alan that got 86'd from Telicthoughts by Mike Gene?
    Mike rarely gave anyone the boot but for some reason you got it.

    ReplyDelete
  17. As regards to Rituals it doesn't help when Priests dumb down the Holy Sacrifice.

    ReplyDelete
  18. I guess I don't see how critics of religion can be faulted for not knowing "what the religions they criticize actually believe" when the religions themselves don't unambiguously articulate the belief.

    Take the recent stuff on "original sin." Seems like there are several possible ways of construing what it is and how it might reconcile with a currently prevailing picture of human origins. No way, from what I read, was presented as the one way officially and unambiguously endorsed by Religion X, Y, or Z.

    The ways of construing original sin all seemed to recognize that the doctrine did not necessarily and unambiguously derive from scripture (hence, why later commentators have felt the need to defend and elaborate on the biblical support). The ways also glossed over whether there really is such a thing as original sin, either as a genuine state of humanity or as a result of a distant ancestor's moral transgression. All the ways seem to accept the doctrine itself without question.

    In the end, what's important to understand is that we are fallen and we are in sin, no matter how it happened we got there.

    JM

    ReplyDelete
  19. Meme said:

    "... got 86'd from Telicthoughts by Mike Gene?
    Mike rarely gave anyone the boot but for some reason you got it."

    Excommunicated!?! Say it ain't so. Because being in "the club" is so important to the religionist

    ReplyDelete
  20. Excuse off-topic response to:

    Alan Fox? Same Alan that got 86'd from Telicthoughts by Mike Gene?
    Mike rarely gave anyone the boot but for some reason you got it.


    As a point of information, Gene had abandoned Telic Thoughts (it was never just his blog according to Mike) before I, along with several other commenters sceptical of ID, were banned there, I think by someone called Nelson Alonzo, AKA guts. It used to be quite a lively place for discussion. Check it out now.

    I never received an explanation for the ban but it coincided with a visit from Paul Nelson, an ID celeb and I did ask him about his "There isn't an ID theory yet" statement.

    ReplyDelete
  21. Is it just me or does anybody else want to make the connection between Dr. Feser's C3 and Walker Percy's Lost in the Cosmos.

    In the book C3 stood for enlightened consciousness (if I am not mistaken).

    ReplyDelete
  22. If you find Fundamentalist A1 Atheism so attractive that is your choice.

    Now, Rock, how have you arrived at the conclusion that I fit into Fesr's A1 slot when I specifically said thay don't work as categories?

    Looking at 1, there are three separate statements and I agree strongly with one of them, I also agree, not strongly with four items in 2, and two strongly in 3.

    ReplyDelete
  23. Alan Fox said:

    "Live and let live, ..."


    Why, does it enhance reproductive success?

    ReplyDelete
  24. Why, does it enhance reproductive success?

    It might, depending on available resources. But that would be irrelevant to why I suggest "live and let live". I'd rather live in a free and open society where a plurality of views are allowed to be aired without rancour than in some fundamentalist-ruled police state. In act, I do, though it's not perfect. Lucky me!

    ReplyDelete
  25. Sorry, touch of arthritis in the fingers,

    S/B in fact

    ReplyDelete
  26. Alan, you were posting under other names there too.
    Don't act like you were a mere skeptic.

    ReplyDelete
  27. Anonymous at 9:41 AM

    Sure, because we all know that PT or ATBC never banned anyone, right?

    ReplyDelete
  28. Sorry for off-topic response.

    Again not correct. A while after I was banned (without explanation, remember, here was a thread on climate denial-ism and I couldn't resist posting a comment. The pseudonym I used was Zykander, I think. Nelson Alonzo spotted my IP adress straight away. The blog had got that quiet that it didn't require much vigilance. So one attempt to comment, one alias. On a blog that had no scruples about censoring inconvenient comments almost as routinely as Uncommon descent. Pandas Thumb and AtBC aren't in the same league. There are moderators with contactable emails if you have a problem. Heck, I bet Ed Feser could post at PT that Jerry Coyne is a vulgar crank and it wouldn't get binned.

    ReplyDelete
  29. Alan you said it yourself you resist the siren call of philosophy.

    Thus you are A1 buy default. That's like saying "I reject science & I only believe the Bible".

    Sorry but your version Atheism is not rational.

    ReplyDelete
  30. I'm tempted to post a link to when Nelson and his side-kick chunkdz put in an appearance at AtBC. They weren't banned.

    Sorry, off-topic.

    ReplyDelete
  31. You can be an Atheist & believe in philosophy. Indeed there is no reason why philosophy must be seen as something that is somehow at odds with Atheism.

    It might be at odds with Positivism & Scientism but those are irrational beliefs on the level of believing in the FSM.

    ReplyDelete
  32. Alan you said it yourself you resist the siren call of philosophy.

    Yes, I should have said religious apologetics. I was too hasty condemning philosophy in general. People reminding me of A J Ayer have set me straight.

    ReplyDelete
  33. Thus you are A1 buy default. That's like saying "I reject science & I only believe the Bible".

    I may be wrong in some of my statements and if it can be pointed out I'll acknowledge my error but I do try to be truthful. It is also fair to say that I know what I think better than you. i don't presume to know what you think without asking, do me the same courtesy and we'll get along.

    ReplyDelete
  34. >Yes, I should have said religious apologetics. I was too hasty condemning philosophy in general.

    Then your stock goes up. That is a rational response.

    ReplyDelete
  35. Alan since you misstated your original view and have corrected it you & I have no beef.

    You accept philosophy you are rational.

    Go in peace.

    ReplyDelete
  36. I think Dawkins, Coyne and myself are all B2's (perhaps Dawkins has more 1ish tendencies but I think he is clearly a B). I can only describe my own view, but I suspect Coyne and Dawkins would take a similar stance.

    I regard things like the ontological argument and first cause argument as "easily" refuted today, but only in light of the historical development of empirical science. I don't think contemporary theologians are stupid or make simple logical errors (they seem smart and well-read), I just think they try to reason about the world in the wrong way and too easily mistake rationalizations for explanations. Dawkins can confidentally dismiss the ontological argument not because he thinks he is smarter and can apply logic more clearly than theologians, but because this kind of argument has always failed to generate any understanding of the world.

    In 2000 BC (or more arguably 1600 AD), a very smart person might have been completely reasonable to think they could arrive at important substantive truths about nature by logical reasoning alone. We now understand that it is only the combination of theoretical model building and empirical observation that can lead to such truths because the space of possible models is too large to pin down a priori. Theologians believe that their "metaphysical" explanations necessarily underlie the success of modern science, but this is a claim belied by the fact that those explanations fail to generate any useful guidelines for doing science beyond the tenets of naturalism (if theological metaphysics doesn't add anything to our understanding of how to do science then it doesn't explain anything at all, it just repeats or confuses). The question is not whether historically scientists could be motivated by their religious/theological beliefs, but whether today we have any reason to think that a theological metaphysics contributes something beyond a naturalistic one. If Plantinga's advice to Christian scientists leads to a generation of Nobel Laureates whose discoveries are inspired by their rejection of materialism and naturalism, that would be a resounding victory for theology, but I'm not holding my breath.

    Scientists as a whole are remarkably dismissive of theology (e.g. the 93% of the National Academy of Sciences which identifies as atheist or agnostic); theologians (and especially Feser) attribute this to the fact that these scientists are just ignorant of theology. If the scientists only knew of the marvelous investigations of the theologians, they would be compelled by the sheer force of their logic (or at least humbled by the complexity of the questions, a la Quentin Smith). I think this is nonsense. Scientists formulate dense and difficult theories all the time - but if the theory teaches them something new, their colleagues will be motivated to study it further. The best philosophical theories generate predictions in this sense (although the subject matter is often sufficiently difficult that it takes hundreds of years before they can be formulated in a scientific fashion). Scientists are ignorant of theology because theologians don't tell them anything about the world and because theologians tortuously construct their theories to avoid empirical disconfirmation.

    And no, gesturing vaguely towards questions we don't know how to pose in a coherent way also does not count as a victory for theology (consciousness! why is there something rather than nothing!). When scientists develop an understanding of consciousness, it will revolutionize the world and change our lives because it will enable us to do things we could not previously do - this is what a real explanation does - it will not sit largely ignored in a dense book for 750 years recalled only by a few cognoscenti.

    ReplyDelete
  37. @Alan Fox:

    Curious. You cite A.J. Ayer as an example of "good" philosophy. Do you realize that the philosophy Ayer promoted to the English speaking world, logical positivism, is demonstrably incoherent? If you want more information, I will be happy to explain the many deficiencies of this theory to you.

    ReplyDelete
  38. Curious. You cite A.J. Ayer as an example of "good" philosophy.

    I can't judge his quality as a philosopher. What impressed me most was his standing up to Mike Tyson.

    ReplyDelete
  39. ...logical positivism, is demonstrably incoherent? If you want more information, I will be happy to explain the many deficiencies of this theory to you.

    Is Wikipedia right in stating logicalpositivism is also known as logical empiricism and scientific philosophy? Sounds as if there might be some worth to the concept. If it's no trouble you could start with the worst deficiency.

    ReplyDelete
  40. Wikipedia says:

    He still believed in the viewpoint he shared with the logical positivists: that large parts of what was traditionally called "philosophy"– including the whole of metaphysics, theology and aesthetics– were not matters that could be judged as being true or false and that it was thus meaningless to discuss them.

    If this is a fair assessment, I can see why you (Untenured) might not be too enamoured. :)

    ReplyDelete
  41. Whoa.

    Someone's mad.

    MAD

    MAD.

    ReplyDelete
  42. ...much of Ayer's book was negative, emphasizing the consequences of a strict application of the positivist program to human pretensions at transcendental knowledge. Traditional metaphysics, with its abstract speculation about the supposed nature of reality, cannot be grounded on scientific observation, and is therefore devoid of significance. For the same reason, traditional religious claims are meaningless since it is impossible to state any observable circumstances under which we could be sure—one way or the other—about their truth. Even much of traditional epistemology is likely to fail the test; only the psychological study of observable human behavior regarding beliefs will remain. Mathematics and natural science are secure, but little else remains.

    A fair description of "Lanuage, Truth and Logc"?

    ReplyDelete
  43. This comment has been removed by the author.

    ReplyDelete
  44. Ah! So some people equate positivism to "scientism". Odd that I should be drawn Ayer by his biographical details and it turns out that his positivism is sometimes pejoratively referred to as "scientism".

    ReplyDelete
  45. Bravo Jason.

    Couldn't have said it better myself.

    ReplyDelete
  46. Untenured said...
    @Alan Fox:

    Curious. You cite A.J. Ayer as an example of "good" philosophy. Do you realize that the philosophy Ayer promoted to the English speaking world, logical positivism, is demonstrably incoherent? If you want more information, I will be happy to explain the many deficiencies of this theory to you.

    In case you missed my earlier reply,

    Yes, please!

    ReplyDelete
  47. I think Dawkins, Coyne and myself are all B2's (perhaps Dawkins has more 1ish tendencies but I think he is clearly a B).

    We're talking about the same Richard Dawkins who has compared the teaching of religion to children as "child abuse," correct?

    Dawkins should be classified as A2 -- he has contempt for religion if held in a modern-day context, since he believes it is both outmoded and harmful, his progressive vision of humanity implying the inseparability of harm from the outmoded.

    ReplyDelete
  48. Excellent post. I'm an atheist and would pretty much class myself as a C3 on your scale ("pro-faith atheist" seems to be the current label and I wouldn't disagree with it).

    I think that religion taps into (and reinforces) a whole host of important psychological and social needs in a way that's pretty much impossible to do without a notion of God or gods.

    I am, frankly, REPULSED by the likes of Dawkins and his Scientism (which is nothing more than declaring God to be dead and then dressing up in His clothes - Dawkins even has his own saints and martyrs: Galileo, Newton, Darwin, etc).

    As I see it, religion is like a stool with a thousand legs. Dawkins breaks one leg and is then infuriated because the stool doesn't topple over. He simply fails to see the role religion actually plays in people's lives and instead views it as a kind of rubbish attempt at science. The man is culturally obtuse and the sooner we can get people to stop listening to him the better.

    ReplyDelete
  49. Philip Cartwright:

    The man is culturally obtuse and the sooner we can get people to stop listening to him the better.

    Send him to Coventry, eh!

    ReplyDelete
  50. I regard things like the ontological argument and first cause argument as "easily" refuted today, but only in light of the historical development of empirical science.

    That's like saying "science refutes haute cuisine." If the ontological argument is 'easily refuted,' someone forgot to tell Kurt Gödel.
    http://plato.stanford.edu/entries/ontological-arguments/#GodOntArg

    contemporary theologians ... too easily mistake rationalizations for explanations.

    If so, it is a crowded field, nor are they the most subject to it.

    this kind of argument has always failed to generate any understanding of the world.

    Of the natural world, I suppose. But was it supposed to? There is a weird unexamined subtext that claims every human activity is really a failed attempt to do SCIENCE!

    ReplyDelete
  51. @Jason:

    "...if theological metaphysics doesn't add anything to our understanding of how to do science then it doesn't explain anything at all, it just repeats or confuses..."

    Jason, I find this puzzling. Why is it that the only measure of the explanatory power of something like 'theological metaphysics' is whether it adds something to our understanding of how to do science?

    jj

    ReplyDelete
  52. BenYachov said...

    Alan since you misstated your original view and have corrected it you & I have no beef.

    You accept philosophy you are rational.

    Go in peace.


    And “Live long and prosper” too. I mean, “Why can’t we all just get along?”

    Though it might help if could decide on a workable definition for “evidence” and that there is a dearth of it from the religious, of virtually all stripes.

    And – relative to that and to some anonymous poster’s comment that “In the end, what's important to understand is that we are fallen and we are in sin, no matter how it happened we got there” [“Don’t beat me no mo’, massah”] – that a comment of T.H. Huxley’s has some validity:

    The deepest sin of the human mind is to believe things without evidence.

    ReplyDelete
  53. Jason,

    I recommend reading The Last Superstition. Feser addresses nearly everything you bring up in your post. Further, Feser does not think all those scientists are atheists because they don't know theology but because they don't understand metaphysics. There's a significant difference there that, I think, may address a lot of your criticisms.

    ReplyDelete
  54. TheOFloinn said:

    “contemporary theologians ... too easily mistake rationalizations for explanations.”

    If so, it is a crowded field, nor are they the most subject to it.


    That some scientists also engage in “peddling” “just-so” stories – as the quote of Richard Lewontin by Dr. Feser (TLS; pg 12) argues (somewhat incorrectly interpreted, I might add) – is hardly a justification for letting theologians off the hook for doing the same thing.

    In addition, it might help if you made an honest assessment of the ratio between tangible benefits and “just-so” stories in the “disciplines” of science and theology.

    And, finally, relative to the first cause argument, I noted – and commented on elsewhere on another thread here – that Bill Vallicella is less than impressed with the “soul as subsistent form” – a concept apparently undergirding a number of arguments for God’s existence (at least in the A-T firmament):

    The problem here, in short, is that there is a tension between soul as substantial form and soul as substantial subsistent form. Ontologically, one wants to protest, a form is not the sort of entity that could be subsistent. Necessarily, a form is a form of that of which it is the form. But a subsistent form is possibly such as to exist apart from that of which it is the form. These propositions cannot both be true.

    While Dr. Feser disagreed with that assessment, the very fact that there is a controversy coupled with the fact that there is literally diddly-squat in the way of evidence to adjudicate between those positions does seem to give some credence to the contention that “Religious belief has no serious intellectual content at all.” And arguing logically – using the correct forms – about fantasies hardly seems to improve matters much.

    I mean, how much controversy do you think there is over, say, the mass of the electron? Plus or minus one part in a billion?

    ReplyDelete
  55. I like this analysis, professor Feser.

    I think that many New Atheist types would *claim* to be B2s, but would *reveal* themselves in a discussion about religious topics to be A1s. I don't know how many times I've had the most dogmatic and ignorant New Atheist type tell me, after a monumentally uninformed and amazingly smug comment, "But hey, I'm open to the evidence, of course, and I don't hate 'religion'." So perhaps we need to put what an atheist claims to be over what he comes across as, e.g. B2/A1.

    ReplyDelete
  56. I have not believed in any gods since the age of about 8 or 9. For most of my life the attitude I took to the theoretical side of a religion was (2).

    Then I started reading apologetics.

    I began with Lee Strobel's stuff, which is just pathetic. Then I advanced to Craig's Reasonable Faith, in which Craig admits his own beliefs are immune to argument (although if you are not a Christian you should be persuaded by arguments!)

    Lastly I took up TLS. This experience firmly moved me into the (1) camp.

    Not only are the arguments feeble, they are atrociously so.

    ReplyDelete
  57. Eric said:

    I don't know how many times I've had the most dogmatic and ignorant New Atheist type tell me, after a monumentally uninformed and amazingly smug comment, "But hey, I'm open to the evidence, of course, and I don't hate 'religion'."

    And, pray tell, what was the evidence that you laid on the table in front of him?

    Weights and density of the soul, perhaps? Kinematics of its transmigration? Signed and notarized communications from the “Heavenly Father”? [Speaking analogously, of course.] And all corroborated and agreed to by the some 38,000 Christian sects, not to mention the myriad of other “faiths”.

    ReplyDelete
  58. "Weights and density of the soul, perhaps? Kinematics of its transmigration? Signed and notarized communications from the “Heavenly Father”? [Speaking analogously, of course.] And all corroborated and agreed to by the some 38,000 Christian sects, not to mention the myriad of other “faiths”."

    Well, you nailed the monumentally uninformed and amazingly smug elements; if only you'd signed off with, "but of course I'm open to the evidence," you'd have exemplified my point completelty!

    ReplyDelete
  59. Eric said:

    Well, you nailed the monumentally uninformed and amazingly smug elements; if only you'd signed off with, "but of course I'm open to the evidence," you'd have exemplified my point completely!

    But you’re still evading the question as to what evidence you actually provided. Really isn’t doing much to disprove the contention that “Religious belief has no serious intellectual content at all”.

    ReplyDelete
  60. Well, you nailed the monumentally uninformed and amazingly smug elements; if only you'd signed off with, "but of course I'm open to the evidence," you'd have exemplified my point completelty!

    That you have no evidence, just endlessly circular arguments?

    ReplyDelete
  61. "But you’re still evading the question as to what evidence you actually provided."

    First, I didn't say anything about my providing evidence to such atheists. Perhaps you should have read my post a bit more carefully before jumping in with your favorite criticism. (What's next --you going to saddle me with talking snakes and a six thousand year old universe?)

    Second, I have to say that you've hit on one of my favorite issues. Let me ask you a question to get things started: Would you say that the proposition, "There is no evidence whatsoever for the existence of any sort of god" is true?

    ReplyDelete
  62. Is it possible for one to be an atheist and believe in a religious type morality? By that I mean believe that lust outside marriage is immoral, pornography is immoral (not just unaesthetic but immoral), etc? Are there atheists that believe in such rules? thanks.

    ReplyDelete
  63. "That you have no evidence, just endlessly circular arguments?"

    What, another one?

    Are you guys plants? You know, theists who pretend to be atheists while commenting on sites like this to make atheists look foolish?

    Where have I said anything on this thread about evidence and arguments I provided to anyone?

    Am I the only one who finds this farcical?

    ReplyDelete
  64. "There is no evidence whatsoever for the existence of any sort of god"

    First we'd need to spell out philosophically what evidence is, i.e., what it means to say that "A is evidence for B."

    ReplyDelete
  65. "First we'd need to spell out philosophically what evidence is, i.e., what it means to say that "A is evidence for B."


    Well, we don't have to do it first, but that's certainly where I was going.

    ReplyDelete
  66. "Well, we don't have to do it first, but that's certainly where I was going."

    Let me clarify: I've found that the people who tend to say things like, "there's no evidence at all for the existence of any kind of god" have not thought seriously for a moment about the necessary and sufficient conditions of evidence, and hence have no clear idea of just what it is that they're saying. That's why I said the analysis doesn't have to come first, but I should have said it isn't necessarily a dialectically prior consideration, though it is an analytically prior consideration.

    ReplyDelete
  67. There are several comments talking about the idea of 'evidence for the existence of God.' I would like Ed's comment on this if he can, but it seems to me that using a term like 'evidence for the existence of God' itself prejudices the answer. It seems to me to presuppose a coherent framework of reality - rationality, in particular, and individual facts within that reality which might point to the existence of some other entity, which we call 'God.'

    But it seems to me that any 'God' that would be be established by such argument could only be Himself something subordinate to that reality which is already supposed obviously to exist.

    And such an entity would not be God as theists understand the term.

    Perhaps I am missing something here.

    Ed?

    jj

    ReplyDelete
  68. Eric said:

    First, I didn't say anything about my providing evidence to such atheists. Perhaps you should have read my post a bit more carefully before jumping in with your favourite criticism.

    You gave some indication of having had a discussion with some prototypical “ignorant New Atheist type” who was supposedly indicating some willingness to view whatever evidence you had. Since you seemed to be suggesting he wasn’t at all interested in looking at that evidence, I simply asked what was it that you had provided.

    Second, I have to say that you've hit on one of my favourite issues. Let me ask you a question to get things started: Would you say that the proposition, "There is no evidence whatsoever for the existence of any sort of god" is true?

    I would say that you can draw any number of lines between any number of facts and call that god. As Steven Weinberg argued, you can call energy god and find it in a lump of coal, but that hardly adds much heat or light. Seems to me that the question hangs on a definition of god and what would constitute evidence; I certainly don’t see any evidence for a Judaic-Christian one – either of the “classical theist” or the “theistic personalist” breed – though one might make a case for a panentheistic / panpsychic one but that still seems a stretch.

    But you’re the one making the claim – you’re the one who has to justify it - starting with the definition of both "god" and "evidence", along with some supposed testable, empirical consequences if the hypothesis were true.

    And, while you’re at it since it seems apropos, and speaking of the lack of “intellectual content”, maybe you can address this point from the Wikipedia article on metaphysics:

    Rudolf Carnap, in his book “Philosophy and Logical Syntax”, used the concept of verifiability to reject metaphysics:

    “Metaphysicians cannot avoid making their statements nonverifiable, because if they made them verifiable, the decision about the truth or falsehood of their doctrines would depend upon experience and therefore belong to the region of empirical science”.

    ReplyDelete
  69. @Steersman:

    "Rudolf Carnap, in his book “Philosophy and Logical Syntax”, used the concept of verifiability to reject metaphysics:

    “Metaphysicians cannot avoid making their statements nonverifiable, because if they made them verifiable, the decision about the truth or falsehood of their doctrines would depend upon experience and therefore belong to the region of empirical science”.

    He used this argument to reject metaphysics?? How strange! Metaphysics is not empirical science. How can stating the obvious - that metaphysics is not empirical science - constitute a rejection of metaphysics? It certainly constitutes a statement of lack of interest in metaphysics, but I fail to see how it constitutes a rejection of it.

    ReplyDelete
  70. John Thayer Jensen said:

    ... but it seems to me that using a term like 'evidence for the existence of God' itself prejudices the answer. It seems to me to presuppose a coherent framework of reality - rationality, in particular, and individual facts within that reality which might point to the existence of some other entity, which we call 'God.'

    That might be a reasonable philosophical starting point, particularly if one wanted to exhaust all of the conceivable possibilities (e.g. God created the universe last Thursday). Although that does seem like, in the words of our patron Saint – Richard Dawkins (blessings be upon his name), bending over backwards to positively supine lengths.

    Seems to me we all work from the assumption that in fact what we perceive, think, sense, and feel actually comprises a “coherent framework of reality” – more or less, to a first approximation. And since we do seem to survive rather well under that assumption – maybe we’re just disembodied brains in a vat? – it seems that we have to proceed on that basis until we find some evidence to suggest that that is not the case. Do you have any evidence of that nature?

    ReplyDelete
  71. @Steersman:

    Seems to me we all work from the assumption that in fact what we perceive, think, sense, and feel actually comprises a “coherent framework of reality” – more or less, to a first approximation

    Yes, certainly. But it seems to me that when you then look for 'evidence' you are looking for evidence that might point to something within that framework - and whatever that 'something' might be, it wouldn't be God. In the very nature of the idea of God, the 'evidence' cannot be empirical, though empirical things - things like the existence of rationality, natural law, right and wrong, beauty, all imply a Source for these things. Sorry, it is probably just the word 'evidence' that I am concerned about. It seems to presuppose a God within the framework of reality - as opposed to God Who is the Source of that framework and therefore not part of it. Maybe I am just moaning about a word.

    ReplyDelete
  72. I'm looking for a recommendation for a book that explores the Christian experience from a deeply personal, non-political, non-evangelistic, and thoughtful way. I'm not looking to be saved -- I'm well beyond that. And I'm not interested in sentimentalism. I prefer something that develops a sense of awe -- but not a Jonathan Edwards fear and trembling. Fiction or non-fiction, any century. I ask this question in all sincerity.

    ReplyDelete
  73. John Thayer Jensen said:

    He used this argument to reject metaphysics?? How strange! Metaphysics is not empirical science.

    My god! I think you guys have been cooped up in the ivory tower too long and have lost sight of the fact that your premises are highly provisional and contingent even if your logic might be impeccable (though even that is debateable).

    Dr. Feser, showing again at least some intellectual honesty, clearly laid out the basis for his argumentation:

    Geometrical reasoning, and mathematical reasoning in general [and, by extension metaphysical reasoning?], is all-or-nothing. The premises are indubitable, and in any argument that appeals to them, the conclusion either follows necessarily from those premises ... or it does not follow .... In general, the starting points of metaphysical arguments aren’t matters of scientific controversy, but rather premises concerning that which science, like common sense, necessarily takes for granted. [TLS pgs 82,83]

    But the premises are anything but “indubitable”, anything but "granted" and the “common sense” basis for them is highly questionable – as any number of visual and auditory and perceptual illusions will attest. For example, as mentioned earlier, Bill Vallicella is less than impressed with the idea of “substantial subsistent forms” on which the whole edifice of God and the immortality of the soul is predicated.

    Unless you can show that those premises are actually true statements about reality then, IMHO, your whole metaphysics collapses like the proverbial house of cards – a serious lack of “intellectual content”, indeed. That, I think, was Carnap’s point, and an eminently sensible one.

    ReplyDelete
  74. "I'm looking for a recommendation for a book that explores the Christian experience from a deeply personal, non-political, non-evangelistic, and thoughtful way. I'm not looking to be saved -- I'm well beyond that. And I'm not interested in sentimentalism. I prefer something that develops a sense of awe -- but not a Jonathan Edwards fear and trembling. Fiction or non-fiction, any century. I ask this question in all sincerity."

    Christian mysticism?

    http://www.amazon.com/Essential-Writings-Christian-Mysticism-Classics/dp/0812974212/

    ReplyDelete
  75. Steersman,

    I think your analysis is correct. At least in TLS, Feser's premises are either empirically false or simply unverifiable assertions. Dubious ground indeed.

    But nothing in that book was as bad as his recent theory about a god injecting souls into Adam and Eve, and then adding the cherry "And there is no evidence against this supposition."

    What could possibly be evidence against such a claim?

    I have undetectable gremlins dancing on my head. There is no evidence against this supposition.

    ReplyDelete
  76. There is no evidence against this supposition.

    Sure there is. Just wave your hand over your head and see if it bumps up against anything. If it doesn't, then that fact constitutes evidence for there being no gremlins on your head.

    ReplyDelete
  77. John Thayer Jensen said:

    Yes, certainly. But it seems to me that when you then look for 'evidence' you are looking for evidence that might point to something within that framework - and whatever that 'something' might be, it wouldn't be God.

    Now, that seems like a reasonable argument and, I suppose, from the few bits and pieces I’ve picked recently up on the “problem of induction” – some from Stove, raises the question to what degree one might infer something from a set of things or facts that don’t entail any facet of the conclusion one is trying to infer or justify – though that seems questionable as one might ask then how one could have come up with the conjecture in the first place. But it seems that all one can do is make some hypothesis and then see what type of consequences might follow from the supposed truth of that conclusion. But that is, if I’m not mistaken, the essential aspect or feature of the scientific method, the “hypothetico-deductive scheme of scientific reasoning”, in the words of the British scientist and Nobel laureate P.B. Medawar [Interesting reading his “Art of the Soluble”; highly recommended.]

    But it still seems entirely invalid to dogmatically assert the existence of some hypothetical entity without the least shred of evidence as backup.

    In the very nature of the idea of God, the 'evidence' cannot be empirical, though empirical things - things like the existence of rationality, natural law, right and wrong, beauty, all imply a Source for these things.

    Again, you seem to be presupposing the existence of that entity. That is maybe of some utility if it leads to some hypothesis about what might follow in this more benighted sphere if that entity actually exists. But one cannot, just by hypothesizing something leap to the dogmatic assertion as to its existence. Not unless you wish to dispute the aphorism, “If wishes were horses then beggars would ride”.

    Sorry, it is probably just the word 'evidence' that I am concerned about. It seems to presuppose a God within the framework of reality - as opposed to God Who is the Source of that framework and therefore not part of it. Maybe I am just moaning about a word.

    That “framework of reality” phrase seems to betray a highly questionable premise, a serious contradiction in terms somewhere along the line, as if to say that God, if He/She/It actually exists, is not real, is not part of the natural order. Part of the problem in this discussion is, I think, related to the definitions and connotations of the words “natural” and “supernatural” with the latter, in my view, being an oxymoron, like married bachelor. Really an incoherent concept to begin with and a place where the whole conversation tends to go off the rails. Seems that whatever “exists” – and that is a problematic word in itself – is still natural, whether all that it encompasses is tractable, definable, perceivable or quantifiable by science or not.

    ReplyDelete
  78. BeingItself said (this is relative to your first post; running out of time here):

    Lastly I took up TLS. This experience firmly moved me into the (1) camp.

    Not only are the arguments feeble, they are atrociously so.


    I tend likewise to the (1) camp, although I’ll argue that there is still some wheat in amongst the chaff, even if that is largely due to its Aristotelian roots. Sort of like arguing that Hitler was Ok because he, supposedly, loved his mother. Though, on the other hand, one might argue that the dissemination of that tree and that particular benefit of the fruit were due, in part, to the efforts of the Church.

    And likewise with the feebleness of the arguments. Seems to me, though it’s difficult to disentangle the metaphysical bafflegab, that the supposed caricature of the cosmological argument so fondly and enthusiastically thrashed by the “Gnu Herd” is not all that far removed from that championed by Aquinas and Feser:

    … the world wouldn’t exist here and now, or undergo change or exhibit final causes here and now unless God were here and now, and at every moment, sustaining it in being, change, and goal-directedness [TLS; pg 86]

    Supposedly God is there as the fellow behind a myriad of sticks pushing, in an “essentially ordered causal series” [pg 107] (analogously speaking), every last quark and gluon to be all that it can be. But I find that not terribly different from a God who simply “knocked down the first domino” [pg 86]. Reminds me of something I recently ran across in a book on logic (Graham Priest; Logic – A Very Short Introduction; Oxford University Press):

    This is one version of an argument for the existence of God, often called the Cosmological Argument. One might object to the argument in various ways. But at its heart, there is an enormous logical fallacy. [shortening, a bunch of predicate logic, followed by:] But if everything has some cause or other, it does not follow that there is one thing and the same thing which is the cause of everything. (Compare: Everyone has a mother; it does not follow that there is someone who is the mother of everyone.) [pg 22]

    ReplyDelete
  79. "But if everything has some cause or other, it does not follow that there is one thing and the same thing which is the cause of everything."


    You know, if a retarded pigeon happened to stumble across Feser's blog, even it could quickly figure out why this objection is intellectually worthless.

    ReplyDelete
  80. Anonymous said:

    You know, if a retarded pigeon happened to stumble across Feser's blog, even it could quickly figure out why this objection is intellectually worthless.

    Well then, if it takes that few neurons to see the holes in the argument then you shouldn’t have any difficulty in providing the explanation – and logical derivation – why that is the case. Followed up, of course, with the correct, intellectually valuable argument along with some evidence to justify why any of the premises used actually correspond to reality.

    ReplyDelete
  81. Well then, if it takes that few neurons to see the holes in the argument then you shouldn’t have any difficulty in providing the explanation – and logical derivation – why that is the case. Followed up, of course, with the correct, intellectually valuable argument along with some evidence to justify why any of the premises used actually correspond to reality.

    I don't know about retarded pigeons, but there was a whole series of posts on this issue a few weeks back.

    Quantifier Shift Fallacies

    ReplyDelete
  82. @jj,

    I am using science in a very broad sense here, but if you concede that theological metaphysics adds nothing to our understanding of science, what is it exactly that you think it does add to our understanding of? Please tell us the particular true claim about anything in the world that we would not know without theological metaphysics.

    @Pattsce,

    I've read parts of TLS and find what I have read polemical and unenlightening. I appreciate the distinction between theology and metaphysics. The problem as I see it is that Feser and fellow theologians place far too much faith in the power of metaphysical reasoning. What has metaphysical reasoning accomplished? My view is that 99.99% of what is written under the guise of metaphysics is wrong or incoherent. The other .01% ultimately becomes incorporated into scientific practice. This is not to say that no one should do metaphysics, only that any conclusion reached by appeal to a particular metaphysical doctrine rather than established science is almost certainly wrong. If you try to use metaphysics to determine the nature of consciousness, the origin of the universe or the age of the earth, you will fail. Feser and other theologians would give metaphysical reasoning a privileged status it does not deserve.

    ReplyDelete
  83. This whole combox conversation, makes me miss smart rational atheist like Dguller. honestly, Dguller would either ask very relevant question or proffer and objection that actually made you think.

    on a (perhaps) positive, note, Djindra is actually asking for help (in apparrent sincerity)without his usually trollish MO. Wonders shall never end.

    ReplyDelete
  84. >>Jason:What has metaphysical reasoning accomplished?

    Metaphysical reasoning is absolutely impossible to avoid. Our only choices are between doing it badly and doing it well. In fact, even the question of what the status and credentials of metaphysics are falls within the jurisdiction of metaphysics.

    >>Jason:"if you concede that theological metaphysics adds nothing to our understanding of science, what is it exactly that you think it does add to our understanding of?"

    The fundamental structure of reality.


    But if you're the sort of person who requires specific examples: The nature of identity, the nature of change, the nature of necessity, the nature of possibility, the nature of essences, the nature of causation, the nature of counterfactuals, the nature of agency, the nature of actions, the nature of events, the nature of space, the nature of time, the nature of universals, and the nature of particulars.

    And if you're the sort of person who is intellectually honest about a subject you evidently aren't sufficiently familiar with, then here is a good introductory text to disabuse yourself of the oxymoronic notion of a "metaphysics-less view of reality":

    http://www.amazon.com/Survey-Metaphysics-J-Lowe/dp/0198752539/

    ReplyDelete
  85. Untenured said...
    @Alan Fox:

    Curious. You cite A.J. Ayer as an example of "good" philosophy. Do you realize that the philosophy Ayer promoted to the English speaking world, logical positivism, is demonstrably incoherent? If you want more information, I will be happy to explain the many deficiencies of this theory to you.

    In case you missed my earlier reply,

    Yes, please!

    ReplyDelete
  86. BeingItself said:

    But nothing in that book was as bad as his recent theory about a god injecting souls into Adam and Eve, and then adding the cherry "And there is no evidence against this supposition."

    Guilty until proven innocent.

    Interesting clash of philosophies though. And interesting difference between that and the last time the Church tangled with science in a serious way (Galileo) with the difference being the question of conflict with “infallible teaching” or not:

    After all, the doctrine is hardly incidental. It is de fide -- presented as infallible teaching -- and it is absolutely integral to the structure of Catholic theology.

    It will at least be interesting to see if the Church can get out of the corner it’s painted itself into.

    ReplyDelete
  87. Hi Steersman,

    When you clearly quoted Dr Feser's TLS, I'm wondering how you could have missed on the same pages (a) the answer to your parenthetical question, and (b) the response to your comment:

    "But the premises are anything but “indubitable”, anything but "granted" and the “common sense” basis for them is highly questionable – as any number of visual and auditory and perceptual illusions will attest." (emphasis mine)

    Quoting from TLS, p. 82:

    They are also like geometrical arguments in being all-or-nothing, though unlike geometrical arguments in that they take empirical premises, rather than pure abstractions, as their starting points.

    And from TLS, p. 83:

    And the empirical starting points are always so general that there is no serious doubt of their truth: for example, they will be premises like "More than one object is red," or "We observe change going on in the world around us."

    Rather than railing about people cooped us in ivory towers, please consider that the evidence you're demanding of others is all around you but you're blind to it, like you were the words in the book, for some reason.

    Maybe it's your prior commitment to atheism and wikipedia.

    ReplyDelete
  88. xerces,

    I wouldn't go quite so far as to advocate a "metaphysics-less view of reality." What I said above - and what I maintain - is that the vast majority of what is called metaphysics is nonsense. This view is not confined just to Feser and Plantinga, but also to people like Quentin Smith (who might reach opposite conclusions, but in my view similarly gives undue credence to the reliability of metaphysical reasoning). The choice is not between doing metaphysics well and doing it badly. Everyone does metaphysics badly because no one knows how to do metaphysics. It is simply an unreliable mode of reasoning which consists of pontificating about questions we don't yet know how to properly ask. In the very long-run, metaphysics may make useful contributions to knowledge, but anyone who studies metaphysics will spend the vast majority of their time writing about nonsense.

    What does this imply? Only this: that expertise in metaphysics does not make it more likely that you will be able to evaluate claims about the world. It does not make it more likely you will be able to evaluate whether the universe has existed forever or began at a finite point in the past, it does not make it more likely you will be able to determine the ultimate fate of the universe, it does not make it more likely you will be able to determine the age of the earth, the nature of consciousness, the nature of ethical or moral claims or anything else that metaphysicians commonly write about. The relevant expertise to answer these questions either comes from science or it does not exist.

    The answers you give for "knowledge" that comes from metaphysics are a perfect case in point. Many of the examples you mention remain vague and ill-defined despite the best efforts of philosophers to clarify them and I am quite confident that in no case can you point to any claim about any of the issues you list that could be called both a subject of consensus among philosophers who have studied the matter and not trivially true (by "trivial" here, I mean claims that follow immediately from the law of non-contradiction once terms are properly defined).

    Finally, it's important to point out that I'm not just articulating this view as an outsider who distrusts philosophy (as if the scientists sit in one camp and the philosophers on the other, each thinking the other inexcusably ignorant). My view that almost everything that has been written on the subject of metaphysics is false or nonsense is one held by many historical and modern philosophers. Scientists have actual knowledge about the world because they have gained expertise in the procedures which reliably lead to such knowledge. Metaphysicians have no such expertise. You will undoubtedly claim that metaphysicians are still more expert than scientists at asking whether God exists because this is a metaphysical question, but that is part of what is in dispute (and whether it is a metaphysical or scientific question is itself a metaphysical question). To the extent that it is a scientific question, scientists are better placed to answer it. To the extent that it is a metaphysical question, it is unanswerable and probably poorly posed in its current form.

    ReplyDelete
  89. To the extent that it is a metaphysical question, it is unanswerable and probably poorly posed in its current form.

    Or as Rudolph Carnap said:

    "Metaphysicians cannot avoid making their statements nonverifiable, because if they made them verifiable, the decision about the truth or falsehood of their doctrines would depend upon experience and therefore belong to the region of empirical science. This consequence they wish to avoid, because they pretend to teach knowledge which is of a higher level than that of empirical science. Thus they are compelled to cut all connection between their statements and experience; and precisely by this procedure they deprive them of any sense."

    ReplyDelete
  90. @djindra,

    Some books:
    C.S. Lewis, Surprised by Joy
    Thomas Merton, The Seven-Story Mountain
    An interesting (in my opinion) contemporary Catholic mystic is Bernadette Roberts, The Experience of No-Self, The Path to No-Self

    ReplyDelete
  91. Steersman wrote:

    "Unless you can show that those premises are actually true statements about reality then, IMHO, your whole metaphysics collapses like the proverbial house of cards"

    - and as you're cheering the collapse of hated metaphysics, it slips your notice that natural science is obliterated with it.

    Maybe you think it's dubious that there is a physical world existing independently of our minds. Perhaps you find it highly questionable that objective laws of logic and mathematics apply to the objective world outside our minds.

    You may not know that empirical science proceeds from assumptions such as these. You might dislike that they embody metaphysical assumptions. By all means maintain they're wrong - but you ought to be aware how much you undermine science in doing so.

    ReplyDelete
  92. Hello Jason,

    Thanks for your comments. I'm afraid they are mostly at too high a level of generality usefully to address. E.g. when you make sweeping assertions like "99.99% of what is written under the guise of metaphysics is wrong or incoherent," I don't know how anyone is supposed to respond to such a claim without essentially repeating arguments of the sort I set out at length in TLS (or, if you are unwilling to overlook the polemics, in my non-polemical book Aquinas), then waiting for you to give a specific criticism of some specific argument and then go from there.

    I would say at least this much about your comments here, though. First, it's very hard to take seriously the suggestion that guys like Dawkins and Coyne are anything but A1 atheists. Let's take the 1 part -- leaving the A part aside for brevity's sake. Consider the criticisms Dawkins and Coyne make of thinkers like Aquinas (e.g. in Dawkins' The God Delusion). If those criticisms were correct, they could have been made by an eight-year-old at any time in history, because they are extremely obvious criticisms to make of the arguments as Dawkins and Coyne (mis)understand them; we hardly needed to wait for modern science to come up with them. Now Coyne and Dawkins clearly regard Aquinas as representative of the best of religious thought. But in that case it is hard to see how they can take anything but attitude 1 toward the theoretical side of religion.

    Now in fact their criticisms of Aquinas are completely puerile. All Aquinas scholars knows that, and they are, quite obviously, the people who are in the best position to know it -- and I don't just mean people who agree with Aquinas. You'll find lots of people who know Aquinas and medieval philosophy well yet who don't think arguments like the Five Ways work, but who would still say that people like Dawkins don't know what the hell they are talking about and don't present serious criticisms. So there's no way to dodge the point by accusing people of having vested interests. It's a straightforward question of whether Dawkins and Coyne have done their homework. And no Aquinas scholar would agree that they have. And the point is that Dawkins and Coyne are not just attitude 1 atheists, but that they exhibit the kind of ill-informed unreasonableness that I have said is typical of A1 type atheists.

    I would say that a willingness to admit this is really a telltale sign of whether an atheist is serious. If a guy is willing at least to admit "Yes, fine, I agree that Dawkins' attack on the Five Ways is a piece of crap. Let's move on to the more serious issues I have with the arguments," then I know that I'm dealing with someone who is both (a) well-informed and (b) honest. As far as I am concerned it is simply not possible for an atheist to be both and at the same time to refuse to do that much. And yet I find that very few atheists I have engaged with are willing to do that. What else can I conclude but that they don't know what they are talking about, and aren't willing to be honest? And frankly, it is silly and unreasonable for someone to whine when I am harsh with such people. They are asking for it, just as much as creationists who refuse to learn what Darwinians really say are asking for it.

    (continued)

    ReplyDelete
  93. (continued)

    A second response to your remarks I want to make is that it really amazes me how completely unconscious you are of all the questions you are begging. You repeatedly criticize philosophy and theology on the grounds that it does not make empirical predictions, does not facilitate scientific research, and the like -- as if we were all agreed that only empirical science or what contributes in some way to empirical science counted as genuinely rational inquiry. But of course, whether that is true or not is precisely part of what is at issue between defenders of scientism (which is what you are clearly committed to) and their critics. Traditional metaphysicians like me take the view that empirical science makes presuppositions that it cannot in principle justify itself, but which lie in the domain of metaphysics to examine. (Specifically, they lie within the branch of metaphysics known as the philosophy of nature.) The Aristotelian theory of act and potency would be an example of something a metaphysician might argue is presupposed by empirical inquiry. And the key arguments of philosophical theology (such as Aquinas's First Way) are grounded in these sorts of claims -- in claims about what all empirical inquiry itself presupposes, not in specific empirical theories. That is why they don't make "predictions." Not because they are less well established than theories that make predictions, but because they are more well established. For they are grounded in what must be the case in order for any predictive theory even to be possible in the first place.

    I am well aware that you are bound to reject all of this. The point is just that the sort of claims you are making completely miss the point, and fail even to address the claims Thomists and other theists are making. You make assertions to the effect that a serious theory will have to be an empirical one, or contribute in some other way to the progress of empirical science, without realizing that the Thomist is challenging precisely those assumptions about the limits of rational inquiry. The Thomist is saying that rational inquiry is necessarily broader than that, and if you are going to show that he is wrong you need to face that claim squarely and consider seriously the reasons why the Thomist makes it (reasons I have explained in many places, such as in TLS and Aquinas).

    Anyway, I appreciate that you have tried to respond seriously here (unlike some other commenters). I ask you to consider, though, that you have failed to see that you have not really addressed the arguments of the other side, but merely presupposed (without showing) that they are wrong.

    ReplyDelete
  94. Josh said:

    I don't know about retarded pigeons, but there was a whole series of posts on this issue a few weeks back.

    Thanks for the links.

    I’ll take another look in a little more detail later but one or two things leaped out at me:

    There is no doubt that Aristotle, Aquinas, and Locke could be read in the way Gensler and so many others suggest.

    Seems there must be quite a number of people who are less perceptive than “retarded pigeons” yet who still managed, credibly in Feser’s view, to reach the same conclusion that I presented based on the book by Graham Priest, a conclusion that they presumably think is anything but “intellectually worthless”. Who knew that pigeons were evolving so fast?

    Given that these readings are available, and that the writers in question are, as Gensler says, “great minds” who would surely be unlikely to commit so blatant a fallacy, the reasonable conclusion to draw is that they did not in fact commit it.

    That might be commendable to have such faith in the perspicacity if not infallibility of “great minds” from the past but a brief perusal of the history of mankind should show that that is hardly justified. Many discoveries have, of course, stood the test of time but a great many others have gone into the dustbin. Seems a case of special pleading to me ....

    If every reader of this blog owns a computer, it doesn’t follow that there is some one computer that every reader of this blog owns. To think otherwise is to commit what is known as a quantifier shift fallacy. .... If every student in the room owns a pencil, it does not follow that there is a certain pencil that every student in the room owns;

    Yes, well, it seems to me that the “moderate realism” of Aristotle – by which Dr. Feser apparently intends to have his cake and eat it too, and without any consequences to boot – is tantamount to just that:

    The senses observe this or that individual man, this or that individual dog; the intellect abstracts away the differentiating features of each and considers the animality in isolation, as a universal. This is not nominalism, for it holds that universals exist. Nor is it conceptualism, for while it holds that universals considered in abstraction from other features exist only in the mind, it also holds that they exist in the extra-mental things themselves (albeit always tied to other features) and that the abstracted universals existing in the intellect derive from our experiences of these objectively existing things, rather than from the free creations of the mind. [pg 61; my emphasis]

    So, by that token, in the case of the pencils, there is in fact “a certain pencil [the universal, as an “objectively existing thing”] that every student in the room owns” by virtue of the fact of owning a single instance. As that conception of universals seems identical to the quantifier shift fallacy it would seem that one might reasonably argue that there is, as Graham Priest stated quite emphatically, “an enormous logical fallacy at the heart of the Cosmological Argument”.

    ReplyDelete
  95. Professor Feser

    ...the Thomist is challenging precisely those assumptions about the limits of rational inquiry.

    That is the crux of the matter, I guess. Why is it not question begging, though? Is Ayer wrong about metaphysics. What else can we do but choose what to believe on faith? What tools do we have for verifying any fact or claim about the universe other than observation and shared experience?

    ReplyDelete
  96. Apologies, though I doubt Ayer and Carnap differ greatly, I was referring to the Carnap quote I posted earlier.

    ReplyDelete
  97. Quantifier shift fallacies would no doubt be beyond the retarded pigeon's comprehension, but what would not be beyond its comprehension is the following idea, perpetuated ad nauseum on Feser's blog: "Everything has a cause" is a claim that has never, in the history of philosophy, been made by defenders of the Cosmological Argument.

    Ergo, Priest fumbles and crumbles pathetically right from the get-go. He is either culpably ignorant or exceedingly dishonest.

    ReplyDelete
  98. jack bodie said:

    When you clearly quoted Dr Feser's TLS, I'm wondering how you could have missed on the same pages (a) the answer to your parenthetical question, and (b) the response to your comment:

    "But the premises are anything but “indubitable”, anything but "granted" and the “common sense” basis for them is highly questionable – as any number of visual and auditory and perceptual illusions will attest." (emphasis mine)

    They are also like geometrical arguments in being all-or-nothing, though unlike geometrical arguments in that they take empirical premises, rather than pure abstractions, as their starting points.


    Lot of material in the book, not all of it easy to parse or comprehend.

    But I don’t have any problem with various examples – as “evidence all around me” – when there is some tangible aspects to them. What I object to or seriously question are other supposed “empirical premises” for which there is no tangible aspects or correlation and are reached only by, apparently, various inductive leaps of faith. For example, as mentioned and quoted earlier:

    But if universals, propositions, and mathematical [objects] are eternal and necessarily existing entities … [TLS; pg 90]

    Now if the essence of a thing and the existence of a thing are distinct in this way …[pg 104]

    Now if there really are Aristotelian natures, essences, final causes, etc., then the lesson of all this …[pg 145]

    These seem to me to be very clearly supposedly empirical premises for which there is not a shred of tangible evidence or proof to justify them. And Dr. Feser wouldn’t have been prefacing them with “if” if he knew they were self-evident and accepted by all in the field.

    ReplyDelete
  99. Steersman said:

    "And Dr. Feser wouldn’t have been prefacing them with 'if' if he knew they were self-evident and accepted by all in the field."




    Modus Ponens:

    If P, then Q.
    P
    --
    Q


    Modus Tollens:

    If P, then Q
    ~Q
    --
    ~P



    I suppose in Steersman's strange, strange world, MP and MT would have to be chucked out the window into the intellectual trash heap.

    ReplyDelete
  100. Anonymous said:

    I suppose in Steersman's strange, strange world, MP and MT would have to be chucked out the window into the intellectual trash heap.

    You really, really, really should take a close look at what Bill Vallicella is saying relative to my quoted list of “if”s, notably “universals, propositions, and mathematical [objects] (as) eternal and necessarily existing entities”:

    The problem here, in short, is that there is a tension between soul as substantial form and soul as substantial subsistent form. Ontologically, one wants to protest, a form is not the sort of entity that could be subsistent. Necessarily, a form is a form of that of which it is the form. But a subsistent form is possibly such as to exist apart from that of which it is the form. These propositions cannot both be true.

    Saying “If I had a million dollars” does not put that money into my pocket. Not in my world anyway, maybe yours ….

    ReplyDelete
  101. jack bodie said:

    ... and as you're cheering the collapse of hated metaphysics, it slips your notice that natural science is obliterated with it.

    Not at all. Methinks you think your particular brand of metaphysics is the whole ball of wax – rather presumptuous at best. Seems a great many other philosophers engage in the same processes without necessarily buying into the A-T brand. Again, as mentioned several times now, Bill Vallicella for example.

    Maybe you think it's dubious that there is a physical world existing independently of our minds. Perhaps you find it highly questionable that objective laws of logic and mathematics apply to the objective world outside our minds.

    Not quite sure how to parse that. But I think it quite likely that there is in fact such a world “independent of our minds”. Though I’m not sure what you mean by “apply to the objective world” as I think you think those laws have some independent existence and autonomy – consistent with your supposed adherence (dogmatic I would say) to Aristotelian realism. My view is that they are only, or largely, models, abstractions, representations, shadows on the cave wall.

    You may not know that empirical science proceeds from assumptions such as these. You might dislike that they embody metaphysical assumptions. By all means maintain they're wrong - but you ought to be aware how much you undermine science in doing so.

    Again, you seem to be thinking that just because I reject or question some of the metaphysical assumptions, the hypothetical premises, of A-T that I am rejecting all metaphysical assumptions and arguments. Which is not at all the case.

    ReplyDelete
  102. Steersman

    "But I don’t have any problem with various examples – as “evidence all around me” – when there is some tangible aspects to them"

    Once again, the answers to the problems you have with the quoted parts of TLS are in TLS so I'm not sure if your objections are reachable by reason.

    Anyway, you have Dr. Feser's book - it certainly doesn't diminish on multiple readings so that's probably your best route to undoing your confusions. I expect you'll find:

    The premises you tend to object to are themselves the conclusions of arguments whose premises you would not deny (eg, we observe change going on in the world around us)

    You should consider that when you preface your statements "It seems to me..." you may as well write "No matter what evidence says or logic compels, I will believe..."

    As to the "tangible aspects":

    Consider that a proton is more than an ensemble of tracks in a cloud chamber, and mass is not merely the deflection of a needle on a set of scales.

    The supposedly solid empirical observations that (you say) alone count as evidence depend on further hypotheses, and can in principle have no more certainty than the hypotheses upon which they rest.

    And when these hypotheses assume intangible aspects (hey, numbers would help us out here, and can we have falsifiability without the principle of non-contradiction - I know, let's assume math and logic!) you can't reject metaphysics in the manner of Carnap without dismissing your faith in the scientific evidence you consider strongest.

    ReplyDelete
  103. Steersman

    "Again, you seem to be thinking that just because I reject or question some of the metaphysical assumptions, the hypothetical premises, of A-T that I am rejecting all metaphysical assumptions and arguments. Which is not at all the case."

    You quoted Carnap as rejecting metaphysics, not A-T, and thought enough of his quote that you asked for a response.

    ReplyDelete
  104. Jack Bodie:

    You quoted Carnap as rejecting metaphysics, not A-T, and thought enough of his quote that you asked for a response.

    You may be mixing Steersman's up with mine. I quoted Carnap in a comment just before Steersman and am interested in a response.

    ReplyDelete
  105. Alan Fox

    Steersman did quote Carnap on September 15, 2011 7:34 PM, quoting from the wikipedia article.

    I see you also quoted Carnap in support of a point that Jason made. I would say Dr. Feser's second combox reply to Jason is just as apposite to your quote. Carnap is begging the question in assuming the standards of empirical science:

    a) apply to metaphysical argument, and

    b) are somehow more reliable than the proofs that pertain to metaphysical argument

    And the Carnap quote you gave on September 16, 2011 12:11 AM talks a bit about the motivations of metaphysicians but it isn't a question of psychology.

    ReplyDelete
  106. Sorry to have butted in!

    My quote is longer but contains the same passage.

    Would I be better moving on to Daniel Dennett?

    ReplyDelete
  107. "Seems to me that the question hangs on a definition of god and what would constitute evidence; I certainly don’t see any evidence for a Judaic-Christian one – either of the “classical theist” or the “theistic personalist” breed"

    OK, good. Now, could you provide me with an *analysis* -- not a definition (we all know what that is), but an analysis in terms of necessary and sufficient conditions -- of the concept, 'evidence'? However we 'define' god, the understanding of 'evidence' you accept is what you're going to use when considering what is and isn't evidence for his existence, right? So, it seems to me that we have to get clear about that *first*. I'll await your *analysis* (again, please, no mere definitions) before commenting further.

    ReplyDelete
  108. Steersman:

    “Seems a great many other philosophers engage in the same processes without necessarily buying into the A-T brand. [. . .] My view is that they are only, or largely, models, abstractions, representations, shadows on the cave wall.”


    Great, so contra your Carnap quote you don’t reject metaphysics just (parts of?) Aristotelian-Thomism. You described your own interesting view above; here’s what I find interesting about it –- it consists a number of metaphysical claims “for which,” to use your words “there is not a shred of tangible evidence or [tangible] proof to justify them.”

    Now tell me what method you use to defend your viewpoint against the same attack that leads you to reject A-T?

    ReplyDelete
  109. Ed,

    To Jason: "You repeatedly criticize philosophy and theology on the grounds that it does not make empirical predictions, does not facilitate scientific research, and the like -- as if we were all agreed that only empirical science or what contributes in some way to empirical science counted as genuinely rational inquiry. But of course, whether that is true or not is precisely part of what is at issue between defenders of scientism (which is what you are clearly committed to) and their critics."

    OTOH, what amazes me is the gullibility one must accept if we wander away too far from empiricism. It seems to me that science has proven to be an epistemological valid enterprise whereas all other forms of knowledge have yet to show they are on sold grounds. That's why the incessant philosophical and theological arguing leads to an increasing number of sects with none able to offer compelling evidence for its own validity. Science puts people on the moon while philosophers stare at it.

    ReplyDelete
  110. @Alan Fox:

    Logical positivism is moribund for a reason. Ayer promoted a principle of verification according to which the only cognitively significant propositions are either analytically true or empirically verifiable in principle. The principle itself is neither analytically true nor empirically verifiable in principle, and is meaningless by its own criterion. The theory is transparently self-refuting and was abandoned by its own advocates. And this is to say nothing of the many technical semantic and epistemic problems for the theory which further contributed to its demise. This is why most proponents of scientism have given up trying to refute the possibility of metaphysics. Instead, they tend to fall back on pragmatic arguments of the familiar "science works, metaphysics doesn't" variety.

    ReplyDelete
  111. "...they tend to fall back on pragmatic arguments of the familiar "science works, metaphysics doesn't" variety."

    Well. Something either 'works' or it doesn't. I guess another choice is that it's incoherent or meaning less which I find A-T metaphysics to be.

    I'll take what works.

    ReplyDelete
  112. Anonymous @8:36,

    You responded to I my comment that "I have undetectable gremlins dancing on my head. There is no evidence against this supposition" by saying I could wave my hand and not feel them.

    But I said they were undetectable, so that would not be evidence against them. We could play this game all day. Eventually you would just give up and conclude that I was a wacko.

    And that is exactly my response when I read about a god injecting souls.

    Feser is very good at immunizing his worldview against any contrary evidence. He just adds one more layer to his story and calls it metaphysics.

    You guys seem to think having a theory that is an unsinkable rubber ducky is a strong theory. But that is exactly wrong. If your theory explains everything, then it explains nothing.

    ReplyDelete
  113. Untenured,

    "This is why most proponents of scientism have given up trying to refute the possibility of metaphysics. Instead, they tend to fall back on pragmatic arguments of the familiar "science works, metaphysics doesn't" variety."

    It must be so disheartening to be put in a position to have to scoff at something that works, that gets results. Your beef is with an epistemology that actually expects something.

    ReplyDelete
  114. Not really, the Proponents of Positivism/Scientism are like weirdos who run around claiming the Andromeda Galaxy doesn't exist..Why? Because they can't see it under their microscope! Of course you try to 1) explain to them the difference between micro vs macro objects and that they are making a category mistake and 2) you try to explain to them there is more to reality that can't be discovered by only using a microscope.

    Their wacko response? Plead the undeniable success of microscopes in examining micro objects and making micro discoveries and ignore the rest of the argument that microscopes are not exhaustive.

    Gof forbid I could see myself becoming an Atheist. But never a philosophy hating Positivist Gnu'Atheist. I might as well turn off my brain believe ANSWERS IN GENESIS and be done with it.

    Gnu's are stupid. They can't be anything else.

    ReplyDelete
  115. Untenured is right. Even A.G. Flew at the height of his Atheism rejected Logical Positivism as a dead end.

    Sorry guys your like YEC using invalid anti-Evolution arguments based on the science of the 50's arguing against the science of the 90's.

    You are out of date.

    ReplyDelete
  116. Alan Fox said...

    Why, does it enhance reproductive success?

    It might, depending on available resources. But that would be irrelevant to why I suggest "live and let live". I'd rather live in a free and open society where a plurality of views are allowed to be aired without rancour than in some fundamentalist-ruled police state. In act, I do, though it's not perfect. Lucky me!"

    September 15, 2011 10:30 AM


    And it might not. But all in all you prefer vanilla ice cream to strawberry.

    ReplyDelete
  117. Blogger Alan Fox said regarding logical positivism,

    "Sounds as if there might be some worth to the concept. If it's no trouble you could start with the worst deficiency."


    How about the fact that according to the author of the principle manifesto, "it was false"? http://www.youtube.com/watch?v=4cnRJGs08hE&NR=1

    at 6 minutes 34 seconds

    Also:
    See the 1946 second edition preface for an earlier if somewhat more qualified admission of Ayer's that the emotive theory of meaning was inadequate, and in effect false.

    I still recall sitting in class and discussing with the professor whether the theory that value statements had to be meaningless because they were not subject to empirical verification wasn't itself falsified by the fact that many prescriptive statements were merely predictions of consequences, which were themselves thought to be good or bad on other grounds, and which as such were therefore both descriptive and prescriptive. I guess we should have been assigned the Introduction to the Second edition as well as the main text, since Ayer himself said as much, though he tried to save the larger view.

    What the positivist is forced to do, is to demand that say, good health, or human life itself, should be justified as a prescriptive good, on the basis of some further description of an ultimately trans-human state of affairs.


    Thus, they demand that the term "good" must have a meaning anchored in some reference apart from objective human interests, and that if it cannot be so anchored outside of its frame of reference, it must then ultimately merely be a synonym for the phrase "I find X pleasurable", or maybe a just a grunt of satisfaction ...

    But of course the holders of these views don't actually in general live their own principles; and would for example complain using terms like "right" and "wrong", should you start crushing their necks under the heeel of your shoe. And I doubt very much that if you paused for a moment and asked, "Are you just trying to effect a sympathetic emotional reaction in me by pleading that what I am doing is a violation of your right to live?", that they would admit to it.

    You can pretty much observe this phenomeneon yourself merely by referring to a transhumanist or a deconstructive leaning atheist as an "it" to its face. They certainly don't seem to believe the lines they are spouting if their reactions are any indication. They seem to believe that something really wrong and antagonistic in an objectively wrong sense has occurred, and not just a bit of gaucherie. Go figure.

    And of course the fact that they are confused on the meaning of the word rights, doesn't help either.

    ReplyDelete
  118. Blogger Alan Fox said...


    "Apologies, though I doubt Ayer and Carnap differ greatly, I was referring to the Carnap quote I posted earlier."

    September 16, 2011 12:53 AM



    Guess I should have read completely through the thread before replying.

    You seemed such a fan of Ayer ...

    Must admit that for an atheist Ayer had an ease of manner and a self-aware grace and charm - as well as honesty - that many of his disciples commenting on Feser's blog lack.

    ReplyDelete
  119. Re. Ayer's print admission

    The 1946 2nd ed. preface to the much later, 1980's something probably, Dover printing of "Language Truth and Logic".

    ... In case the lingering context wasn't clear.

    ReplyDelete
  120. djindra,

    "OTOH, what amazes me is the gullibility one must accept if we wander away too far from empiricism. It seems to me that science has proven to be an epistemological valid enterprise whereas all other forms of knowledge have yet to show they are on sold grounds. That's why the incessant philosophical and theological arguing leads to an increasing number of sects with none able to offer compelling evidence for its own validity. Science puts people on the moon while philosophers stare at it."

    Empiricism is a philosophy, not science, so if you're going to rely on it you have an issue if you want to argue that empiricism justifies science. Additionally, the "it works" attitude is reflective of pragmatism ... a philosophy advocated by people like Dewey, James and others. Thus, you are relying on an awful lot of philosophy to make your claims about what is reasonable and what isn't. Without that philosophical underpinning, science doesn't get to the Moon, and without philosophy -- and the debates between empiricists and rationalists, since the empiricism science uses is not, in fact, unvarnished, naive empiricism -- science wouldn't even exist or, if it did, wouldn't be anywhere near as effective as it is (no hypothetico-deductive system without the influence of rationalism). So it seems to me to be quite brave to attack that which underpins your entire enterprise.

    "It must be so disheartening to be put in a position to have to scoff at something that works, that gets results. Your beef is with an epistemology that actually expects something."

    It works for what it works for, but someone who accepts that need not accept that it can work for every principle or area of interest. For example, I do strongly disagree that it can be used for doing ethics, and can quite reasonably deny that it is of much use in working out general and abstract concepts. If you don't care about that, fair enough, but that wouldn't give you the right to crtiicize someone else's project because you don't consider it interesting.

    ReplyDelete
  121. @ DNW

    Thanks for the link to Bryan Magee interviewing Ayer. Most enlightening. The subsequent remarks after "Nearly all of it was false" are a little more nuanced. "What survived was the general rightness of the approach"

    I note there a couple more videos. Quite a lot to absorb. Grateful for the information.

    ReplyDelete
  122. @ Untenured

    As anonymous points out "science works, metaphysics doesn't" is hardly a ringing condemnation of Ayer and positivism. You might want to take a look at the video clip DNW linked to. Bryan Magee, the interviewer is a character in his own right. (See his Wikipedia entry).

    ReplyDelete
  123. @Alan Fox:

    I submit that the resort to "science works, metaphysics doesn't" is evidence of a sharp intellectual decline among proponents of scientism over the last 60 years. The logical positivists were in serious error, but if their view had been correct they could have rationally demonstrated that metaphysics has no cognitive significance. The contemporary appeals to pragmatism do not even purport to show this or anything close to it. Instead, they simply propose to ignore metaphysics because it doesn't help us predict and control natural events. And last I checked, ignoring a position is not the same as refuting it.

    Now, if someone wants to argue that a form of inquiry cannot have cognitive significance unless it results in pragmatic benefits of a particular sort, then I'm all ears. Let's hear the argument, and let's see whether it has some merit. The trouble is, I'm not seeing many contemporary proponents of scientism actually giving the arguments. Instead, I'm seeing an awful lot of Dennett-style rhetorical posturing and waxing eloquent about the glories and wonders of science, world without end.

    In other words, scientism has been resting on weaker intellectual foundations following since the demise of logical positivism, not the other way around.

    ReplyDelete
  124. Alan Fox said...

    @ DNW

    Thanks for the link to Bryan Magee interviewing Ayer. Most enlightening. The subsequent remarks after "Nearly all of it was false" are a little more nuanced. "What survived was the general rightness of the approach"

    I note there a couple more videos. Quite a lot to absorb. Grateful for the information.
    September 16, 2011 9:50 AM


    As I recall, he admits three areas of complete conceptual inadequacy and error in their agenda; and another, i.e., the emotive theory of ethics which is inadequate enough to be wrong as well. Though there is some mumbling about how there was some sense in which it was on the right track. Except where it wasn't of course.

    Ok, I broke down and reviewed:

    "True in spirit" he says LOL

    What doesn't work and is admitted as outright false:

    - Verification principle not formulated adequately and never has been since

    - The strong reductionism of Morris Schlick simply doesn't work

    - What is true about its empiricism insofar as it can be said to be true, is composed of "wishy-washy" platitudes that anyone might agree with.

    - The analythic synthetic distinction he relied upon was shown to be false by Quine.

    - his doctrine of conventionalism is no longer defensible says Ayer

    - Logical Positivism doctrine as it might apply to past or historical statements is admitted as false or useless

    - Ayer admits his theory of other minds was wrong

    "In detail very little survives"
    (Very near quote)



    I liked Ayer. He was still alive when I was in college, and I admire his ability to take a critical stance regarding his own work - even if it was largely forced on him.

    But the argument that "our hearts were in the right place" doesn't carry much water.

    Magee may have characterized it better, but still too generously, given the motivations behind the movement.

    If you watch the earlier videos, - which I first saw on the CBC many years ago - you will see Ayer admit that the agenda was basically sociopolitically motivated in the first place.

    If you haven't done so, you should read "A Part of My Life"

    If you have done so, you should read it again, this time slowly and carefully.

    What it reveals through implication about the movement and motivations is more informative than what it explicitly says.

    ReplyDelete
  125. Steersman,

    And Dr. Feser wouldn’t have been prefacing them with “if” if he knew they were self-evident and accepted by all in the field

    This is as good a sign as any that you haven't the intellectual honesty to read a book objectively.

    Assuming that "if" in a philosophical work, is used to convey a sense of hesitancy or reserve is asinine, especially given its place as a logical construct in various forms of arguments (pretty much any argument including an antecedent and consequent). This is evident in modus ponens, modus tollens, hypothetical syllogism, etc.

    Furthermore, the good Dr. Feser obviously doesn't think A-T metaphysics is "accepted by all in the field", whatever field you have in mind. After all, throughout both TLS and Aquinas he addresses arguments against A-T metaphysics and refers to other metaphysical systems prevalent today. Really, I can't figure out how your comment could be based on anything said in his book. Perhaps it was just too difficult to follow.

    ReplyDelete
  126. How metaphysics works:

    1. It is an interesting thing to inquire into, in Sellars' famous phrase, how "things, in the most general sense of the term, hang together, in the most general sense of that term". So to complain that metaphysics "doesn't work" makes as much sense as saying that baseball doesn't work.

    2. But metaphysics is supposedly about learning something, and there is the obvious complaint that one never gets anywhere, that is, to generally accepted conclusions. But the same is the case with political arguments, and the latter are going to go on as long as there is politics. There are various metaphysical approaches, and development within an approach (like A-T, or Middle Way Buddhism, or materialism) does progress as the one approach is critiqued by others or within itself. But what, one might ask, does all this matter? After all, politics does matter for how societies work. Hence,

    3. Metaphysics works to support or tear down one's religious stance (which might be the stance of rejecting all things religious). In studying metaphysics one may strengthen or weaken one or another's religious stance. In the absence of metaphysics, which is to say with no rational discussion, all there is is a shouting match, if not worse. In sum, to engage in rational debate about religious truth, or lack thereof, is to engage in metaphysics. Hence, every commenter here presupposes that metaphysics works.

    ReplyDelete
  127. @SR:

    I like the comment. But it reads more like a dialectical progression. If we want to make it analogous Ed's, it should probably look something like this:

    1)Metaphysics amounts to nothing more than empty verbiage and circular arguments. Metaphysicians come up with arbitrary definitions just so they can arrive at pre-ordained conclusions. Science, on the other hand, really tells what things are like.

    2)Prior to the scientific revolution, it might have made sense for intelligent people to think that metaphysics could establish truths about the nature of reality. Moreover, many intelligent and intriguing claims have been defended by metaphysicians in the past. But, nowadays, we enlightened folk know that the natural sciences suffice to explain everything that needs explaining.

    3)Metaphysics isn't able to establish any truths about reality over and above those established by science. Nevertheless, it takes a lot of philosopical work in order to show that this is true, and an intelligent and scientifically informed person could nevertheless fall for the mistaken belief that metaphysics is a serious form of inquiry.

    ReplyDelete
  128. Anon: "You know, if a retarded pigeon happened to stumble across Feser's blog, even it could quickly figure out why this objection is intellectually worthless."

    What do you mean "if"? They haven't just stumbled across it, they've been posting!

    ReplyDelete
  129. Steersman: "So, by that token, in the case of the pencils, there is in fact “a certain pencil [the universal, as an “objectively existing thing”] that every student in the room owns” by virtue of the fact of owning a single instance."

    Nobody owns the Universals, man. But yeah, the argument as you understand it is nonsense. Fortunately that bears no resemblance to the actual arguments as actual Thomists/etc. understand them.

    ReplyDelete
  130. BeingItself: "We could play this game all day. Eventually you would just give up and conclude that I was a wacko."

    I'm way ahead of you, buddy.

    ReplyDelete
  131. I don't know who this Anon person is but he is stealing my act.;-)

    Cheers!:-)

    ReplyDelete
  132. Anonymous said:

    BeingItself: "We could play this game all day. Eventually you would just give up and conclude that I was a wacko."

    I'm way ahead of you, buddy.


    Except that the empiricist – whether his underlying metaphysics is logical positivism or reformed Buddhism – continues on with the business of making the world tick over: like designing bridges and developing vaccines and building airplanes (as opposed to flying them into buildings which seems the preferred modus operandi of the religious) ....

    ReplyDelete
  133. Steersmen,

    Everybody who has ever developed technology was an empiricist?

    Seriously?

    Every here of the fallacy of the hasty generalization guy?

    ReplyDelete
  134. edit: Ever hear....etc


    I keep getting distracted.

    ReplyDelete
  135. Yeah, Steersy, good point. I believe there was once a philosopher of a pragmatist bent who once said that the goal of philosophy was not to comprehend the world but to change it. Wonder how that philosophy turned out in practice?

    http://www.hup.harvard.edu/catalog.php?recid=26699

    Clearly, it is only religious people who kill in the name of their ideology.

    ReplyDelete
  136. >like designing bridges and developing vaccines and building airplanes.

    A 6 day Young Earth Creationist with either engineering degrees or Medical ones can do all that.

    I don't see what empiricism has to do with it?

    Just like an Atheist can feed the poor or help sick.

    ReplyDelete
  137. @Steersman:

    You write that "the religious" fly airplanes into buildings. Really? I know of some muslims who have flown airplanes into buildings, but I don't know of any Jews, or Christians, or Hindus, or Mormons, or Animists, or Shintoists, or Jainists, or Buddhists who have. Hence, "flying planes into buildings" doesn't sound like the work of "the religious" to me. But keep on speaking in terms of those medacious abstractions, I'm sure it makes you feel smart.

    ReplyDelete
  138. Everybody who has ever developed technology was an empiricist?

    To develop the technology they were using empiricism... unless you can point to an example of technology that was built without it.

    ReplyDelete
  139. BeingItself,

    There is the big problem that gremlins are fictional creations, which constitutes a priori evidence against their existence. If I doodle some sort of fictional creature right now on a piece of paper, odds are overwhelming that it doesn't exist. Darth Vader, Harry Potter, etc., almost certainly do not exist, in virtue of their being creations of fiction.

    There is no comparison between the concept "gremlin" and concepts like "soul" and "God." The latter are backed by philosophical argumentation and are not mere creations of fiction.


    (BTW, if they're completely undetectable, how do you know they're on your head?)

    ReplyDelete
  140. @Anonymous:

    But keep on speaking in terms of those mendacious abstractions, I'm sure it makes you feel smart.

    Apparently it came from Victor Stenger who at least popularized the phrase. So yes, I feel in good company. But no worse a stereotype – quite a bit better I think but maybe I’m biased – than Feser’s:

    For however well-meaning this or that individual secularist may be, his creed is, I maintain (and to paraphrase Dawkins’ infamous description of critics of evolution) “ignorant, stupid, insane, and wicked.” …. As this book will show, reason (???) itself testifies that against the pest of secularist progressivism, there can be only one remedy: Écrasez l’infâme. [TLS; pg ix]

    Rather brave, arrogant and presumptuous words for someone who acknowledges that many other credible individuals and philosophers are less than impressed with Aquinas’ proofs for the existence of the God that supposedly justifies, like the destruction of Sodom and Gomorrah, that particular condemnation.

    Even if all of the religious didn’t fly those planes into those towers, by their adherence to similar fantasies divorced from any obligation or willingness to seek corroboration in reality, they are far more culpable than those who do have recourse to that touchstone.

    ReplyDelete
  141. djindra: "I'm looking for a recommendation for a book that explores the Christian experience from a deeply personal, non-political, non-evangelistic, and thoughtful way."

    I'd recommend a little book called Ben Israel by the late Art Katz.

    From the Amazon webpage:
    Raised in Brooklyn, New York, through the depression years and coming to adolescence during the turbulence of World War II, Art Katz, in his quest for the meaning of life, began a journey toward Truth that climaxed significantly and symbolically in Jerusalem.
    Through the diversity of Marxist, pragmatist, and existential ideologies and philosophies, as well as merchant marine and military experiences, Art was brought to a final moral crisis as a teacher, able to raise, but not answer the groaning perplexities of the modern age and his own heart.

    During a leave of absence, on a hitch-hiking odyssey through North Africa, Western Europe and the Middle East, the cynical and unbelieving atheist - vehement anti-religionist and anti-Christian - was radically apprehended by a God whom he was not seeking. The actual journal, Ben Israel - Odyssey of a modern Jew, recounts the breaking into consciousness and ultimate apprehension of an unsuspecting and resistant 'son of Israel.'

    ReplyDelete
  142. >To develop the technology they were using empiricism...

    No they where using science. Empiricism is a philosophy it is not science.

    If you want to call Philosophy a science that is lovely but then it renders Positivism trivially true.

    ReplyDelete
  143. Anon,

    I could easily construct a feserian type philosophical argument for the existence of my head dancing gremlins.

    The evidence for my gremlins and the evidence for Feser's immaterial god injected soul are exactly the same: none at all.

    But, as Feser hilariously asserts: There is no evidence against either supposition.

    ReplyDelete
  144. Anon,

    Also, by asking for evidence you are acting like an empiricist. Good on ya! You're learning how skeptics think.

    ReplyDelete
  145. There is the big problem that gremlins are fictional creations, which constitutes a priori evidence against their existence.

    What exactly makes them 'fictional'? They may be mythological creatures, but so to are deities.

    There is no comparison between the concept "gremlin" and concepts like "soul" and "God." The latter are backed by philosophical argumentation and are not mere creations of fiction.

    So there can be no philosophical arguments for the existence of gremlins?

    (BTW, if they're completely undetectable, how do you know they're on your head?)

    If the soul is completely undetectable how do you know your have one?

    ReplyDelete
  146. I have seen the following sort of claim many times (as we all have, no doubt):

    "I could easily construct a feserian type philosophical argument for the existence of my head dancing gremlins."

    Such a claim is usually found in the company of similarly -- well, let's just say 'similar' claims, such as, "There is no more evidence for the existence of god than there is for the existence of leprechauns" or "There are no better reasons to believe that god exists than there are to believe that the Flying Spaghetti Monster exists," and so on.

    Beingitself, could you please provide me with that "Feserian type philosophical argument for the existence of [your] head dancing gremlins?"

    It's a funny thing, but whenever a New Atheist type says he can provide such an argument, I ask him to provide it -- and he never does. (Just as New Atheist types can never seem to get around to providing me with an *analysis* of the term 'evidence' when they say things like, "there's no evidence whatsoever for the existence of the sort of god Christians believe exists.")

    So Beingitself, I eagerly await your argument.

    (If I'm just feeding the trolls, I apologize, and promise not to respond when he fails to provide that argument, or provides one that's patently stupid, e.g. parroting the premises for any standard argument for god's existence and substituting 'gremlins' for 'god' in the conclusion, for then he'll have made my point for me. In other words, I'll only respond if he shows himself not to be a troll.)

    ReplyDelete
  147. @Stone Top:

    You write:

    So there can be no philosophical arguments for the existence of gremlins?

    I call bullshit. Show me a single intellectually respectable philosophical argument which purports to establish the existence of Gremlins. Don't just parrot the "empirically unverifiable" trope. Really, show us a serious argument for Gremlins, and show us how it is analogous to Aquinas's five ways or any of the other theistic proofs.

    ReplyDelete
  148. To develop the technology they were using empiricism

    To develop technology they used empirically discovered laws. They also used rationally derived mathematics.

    Empiricism is the doctrine that all truths are either empirically derived or tautologous, that is, vacuously true. Empirical discoveries obviously play a role in science. The doctrine of empiricism does not, at least in any direct way.

    ReplyDelete
  149. On the usefulness of metaphysics to science:

    If one did not believe that everything which arises has a cause, why would one go about looking for one? If one did not believe in the lawfulness and uniformity of nature, one would simply accept anomalous phenomena and not search for an explanation of them.

    ReplyDelete
  150. I could easily construct a feserian type philosophical argument for the existence of my head dancing gremlins.

    What argument would you provide beyond mere stipulation? The existence or non-existence of gremlins (or angels) is an empirical one, to be settled by observation. The existence or non-existence of a necessary creator is a metaphysical one, to be determined by reason.

    ReplyDelete
  151. What exactly makes them 'fictional'? They may be mythological creatures, but so to are deities.

    Because they were whipped up out of the ether of the imagination. Unlike "deities," their existence was in no way established as a conclusion following a strong chain of philosophical argumentation. If I say that there's a planet on the other end of the universe filled with thousand-headed dogs that can breathe fire, then as a matter of probability there almost certainly isn't, in virtue of my having created the idea out of absolutely nothing. Cf. Darth Vader and fantasy art.


    So there can be no philosophical arguments for the existence of gremlins?

    There can be no serious philosophical arguments for the existence of gremlins, conceived as material, Roald Dahl-esque beings.


    If the soul is completely undetectable how do you know your have one?

    Like I said, serious philosophical argumentation that's in keeping with Reason. Reason is an instrument like light, used for illuminating and bumping off of things. Reason is employed for detecting truth.

    ReplyDelete
  152. do you know somebody's selling your book for more than $3,000?

    http://www.amazon.com/gp/offer-listing/1587314517/ref=tmm_hrd_new_olp_0?ie=UTF8&qid=1220302362&sr=1-1&condition=new

    and two others are selling it for $1000.

    ReplyDelete
  153. Anonymous said:

    Yeah, Steersy, good point.

    Yes, well twinkle-toes, I do try. And I also try to quote some relevant passage so my respondent has some hope of finding what I was referring to. Tends to improve my chances of getting a response.

    I believe there was once a philosopher of a pragmatist bent who once said that the goal of philosophy was not to comprehend the world but to change it. Wonder how that philosophy turned out in practice? http://www.hup.harvard.edu/catalog.php?recid=26699

    Thanks for the link; I’ll put it on my reading list.

    Clearly, it is only religious people who kill in the name of their ideology.

    So we can agree that ideology can be a bad thing. Though I might argue that is most problematic in authoritarian societies and cultures where indoctrination is more the rule than the exception .....

    ReplyDelete
  154. Verbose Stoic,

    If I want to be a stickler, empiricism is not a philosophy, it's a theory of knowledge -- a branch of philosophy. Yes, "it works" is somewhat like Pragmatism but let's not forget science predates James and Dewey by a a couple of centuries. I don't think there's much doubt that science sits at the pinnacle of empiricism and in this century there's no good reason to talk of empiricism without referring to science. Science as we know it today is diametrically opposed to rationalism. In no way does it underpin science.

    I agree science is not good at doing ethics. That may not always be true. And I don't know what you mean about science being useless in working out general and abstract concepts. Theories are general and abstract concepts. So I assume you must mean something else.

    ReplyDelete
  155. "I'm looking for a recommendation for a book that explores the Christian experience from a deeply personal, non-political, non-evangelistic, and thoughtful way. I'm not looking to be saved -- I'm well beyond that. And I'm not interested in sentimentalism. I prefer something that develops a sense of awe -- but not a Jonathan Edwards fear and trembling. Fiction or non-fiction, any century. I ask this question in all sincerity."

    djindra,

    Even though I'm Catholic and this novel is told from a more "Protestant" viewpoint, I haven't ever read a better contemporary piece of fiction, filled with Christian themes than Gilead:

    http://old.nationalreview.com/books/hart_200505200838.asp

    ReplyDelete
  156. Daniel Smith, SR, John Thayer Jensen & Anonymous,

    Thanks for the recommendations. They all look like good reads.

    ReplyDelete
  157. Eric said:

    Such a claim is usually found in the company of similarly -- well, let's just say 'similar' claims, such as, "There is no more evidence for the existence of god than there is for the existence of leprechauns" or "There are no better reasons to believe that god exists than there are to believe that the Flying Spaghetti Monster exists," and so on.

    Seems like a perfectly reasonable argument to me. If none of those hypotheses – and that’s what they all are, in my view, at best – provide any benefits, any control, any handles on reality to improve our chances of surviving – as individuals or societies – then they seem like so much dead weight – booster rockets to be jettisoned when empty.

    Maybe you can argue – or do argue – that there are some placebo effects and benefits – and Dr. Feser alluded to some of those in his classification and which I at least sympathize with, humanity’s ability to handle the truth being somewhat limited. But that seems a limited benefit for a rather high cost. Sort of like living in a fool’s paradise ....

    Just as New Atheist types can never seem to get around to providing me with an *analysis* of the term 'evidence' when they say things like, "there's no evidence whatsoever for the existence of the sort of god Christians believe exists.")

    As mentioned earlier, I think you’re the one making the claim – you’re the one who has to justify it. As for evidence of my assertion that you have no evidence I think I can point to the fact that you don’t have any test results showing, say, the efficacy of prayer to a statistically significant level – though “God knows” (sort of), people have tried (unsuccessfully), no results showing an increase in the mass of the Catholic “Host” on consecration, no photographs showing the smiting of any modern day Sodom & Gomorrah (though, God knows, Televangelists from one coast to the other seem to think there’s no shortage of targets), no unambiguously completed prophecies or predictions: nada.

    ReplyDelete
  158. Steersman, again, I see that you've still not provided me with that analysis of the concept 'evidence.' Perhaps I'll get it when Beingitself provides that argument for gremlins.

    ReplyDelete
  159. @Steersman:

    If none of those hypotheses – and that’s what they all are, in my view, at best – provide any benefits, any control, any handles on reality to improve our chances of surviving – as individuals or societies – then they seem like so much dead weight – booster rockets to be jettisoned when empty.

    Now this is definitely getting to interesting stuff. This is about motivation.

    For me, my motivation in thinking about things is to know the truth. I confess to a belief that I myself am not going to survive - in my bodily form, I mean - indefinitely - and - no doubt this is rotten of me, but there it is - I find in myself a feeling that, if, indeed, there is no ultimate absolute meaning to things, then a sense that I really don't care whether society, the human species, or whatever, survives or not.

    I just want to know the truth. And I think that some things that I can know are true are not dependent on empirical tests. This is true about mathematical truths. I think it is true about the existence and nature of God as well.

    Whether it helps me survive (for a little longer) or not.

    jj

    ReplyDelete
  160. Fri 9/16/2011 1:56 PM
    Anonymous said:

    Nobody owns the Universals, man. But yeah, the argument as you understand it is nonsense. Fortunately that bears no resemblance to the actual arguments as actual Thomists/etc. understand them.

    Ok, I’ll send the argument back to the factory for a tune-up. Though, as mentioned, I really think it is a very questionable one – that somehow through the magic and prestidigitation of A-T metaphysics abstractions can take on some autonomous reality and have influence on the course of the universe. And, also as mentioned, it would seem that Bill Vallicella, among others, is also highly skeptical of the premise.

    I can see some sort of remote possibility if, say, there was some quantum mechanical link through entanglement of the neurotransmitters in multiple brains storing the representation for some external object or objects. That might conceivably be testable – but just to assert that entity's existence and its influence without evidence or effort to generate some really doesn’t add a lot to the credibility of the theology.

    ReplyDelete
  161. Djindra--

    I very strongly recommend the book "Meditations On The Tarot", which is subtitled "A Journey Into Christian Hermeticism".

    If you want a thick slice of mystical insight and reasoning, this is it. The first time I read the book, the first 20 or so pages gave me more to think about and cogitate on than all of the books I'd previously read about pantheism and "Eastern Philosophy", and than any book I'd read about Christianity up to that point.

    Half of the battle in deciding for or against theism is getting a glimpse of the potential beauty offered by a rational mystical outlook. Absent this glimpse, it is hard to see why someone wouldn't choose atheism. But with it? Perhaps the book will help to give you this glimpse. I know of none better for this. It is truly sui generis.

    ReplyDelete
  162. Upthread, Steersman stated:

    [But the premises are anything but “indubitable”, anything but "granted" and the “common sense” basis for them is highly questionable – as any number of visual and auditory and perceptual illusions will attest.]

    Of course, to know that there are visual, auditory, and perceptual illusions is precisely to presuppose that our visual, auditory, and perceptions are generally reliable. Otherwise, how could we know that there is such a thing as an illusion? Must not one first know reality and perceive how the illusion fails to reflect reality in order to know that it is an illusion? Does not the existence of counterfeit money presuppose the existence of real money? Does not the existence of lies presuppose the existence of truth?

    ReplyDelete
  163. John Thayer Jensen said:
    Fri 9/16/2011 8:24 PM

    I find in myself a feeling that, if, indeed, there is no ultimate absolute meaning to things, then a sense that I really don't care whether society, the human species, or whatever, survives or not.

    Après moi, le déluge …

    I can sympathize; I think everybody more or less has that in the back of their minds even if they don’t want to admit it – though maybe that’s just a projection. Seems many seek to live on after their bodily deaths one way or the other – through their kids, charitable works, philanthropy, promoting the “greater glory of Man and God” – a great many different manifestations. But all of which tends to keep the ball rolling. And, who knows, if it does keep rolling it might even carry all of us into the Promised Land. So to speak ....

    I just want to know the truth. And I think that some things that I can know are true are not dependent on empirical tests. This is true about mathematical truths.

    A worthy goal, although I question, or wonder at rather, the definition for empirical you have in mind. Hume argued, with some justification I think, that both reasoning and sensing came under the purview of empiricism:

    If we take in our hand any volume; of divinity or school metaphysics, for instance; let us ask, Does it contain any abstract reasoning concerning quantity or number? No. Does it contain any experimental reasoning concerning matter of fact and existence? No. Commit it then to the flames: for it can contain nothing but sophistry and illusion.

    Seems to me that, in some sense, we sense the elements of reasoned argument and mathematics – I remember reading a quote of Einstein in which he was asked how he knew his theories were correct and he said, “I could feel it in my bones”: there’s some more fundamental process or attribute under the hood that ties together those attributes of quantity and number and matters of fact and existence, some fundamental commonality that they share.

    But as Laplace argued in his response to Napoleon, I think God is, at best, a hypothesis, if not a formally undecidable proposition.

    ReplyDelete
  164. @Steersman:

    But as Laplace argued in his response to Napoleon, I think God is, at best, a hypothesis, if not a formally undecidable proposition.

    And although I might quarrel mildly with your language, I would substantially agree. I am morally certain of the existence of God - 'morally' meaning that my certainty is such that I believe I could not be an honest man and not act on it.

    I think more than one has recommended to Djindra the writings of C. S. Lewis - I have, off-list. I generally recommend a trilogy of his - I mean, he didn't write it as a trilogy, but these three books are the ones that I think helped me in the beginning. They are "Miracles," "The Problem of Pain," and "Mere Christianity." The third is the least philosophical of them, and is often - I think mistakenly - where people are told to start. The first two are philosophical - very much matters of popular philosophy, certainly, but by 'philosophical,' I mean they do not depend at all on any idea of revelation.

    And I think Lewis would say - does effectively say in the first two - that moral certainty is what we can have. Logical undeniability perhaps cannot, in the nature of the case, be ours.

    It is certainly possible, without absurdity - in fact, without losing much in the way of plausibility - to deny the existence of God.

    It seems to me, on the other hand, deeply plausible to believe in His existence.

    Plausible - and also desirable, though Lewis didn't see it that way when his first steps in theism came (see "Surprised by Joy"). You may write me down as believing because I want to believe. Quite correct. I think I have good reason to believe - but I have vastly stronger reasons for wanting it to be true.

    Pascal's wager? Perhaps. Faith is, after all, finally an act of the will. Informed by the intellect, absolutely - I am not a fideist! - but finally I could choose to doubt.

    I do not so choose.

    ReplyDelete
  165. Matteo said:

    Of course, to know that there are visual, auditory, and perceptual illusions is precisely to presuppose that our visual, auditory, and perceptions are generally reliable.

    That we manage to survive should be adequate proof of that – at least to a reasonable number of decimal places. I’m reminded of the movie Die Hard 2 in which the bad guys reset the sea level reference for the Instrument Landing System and, as a result, the plane they allowed to land crashed because of that mismatch. If our mental models of reality don’t correspond adequately to that reality then “crash and burn” is typically the result.

    Though it’s curiously interesting how the brain handles that processing. I seem to recollect seeing in the movie What the Bleep Do We Know some fellow talking of an experiment in which they fitted some test subjects with goggles which inverted the scene presented to the eyes. The subjects initially stumbled about because of the inversion but, apparently, after a fairly short time the brain had rewired itself to add an extra inversion to compensate for the one in the goggles and the test subjects could then move about without any difficulties.

    Must not one first know reality and perceive how the illusion fails to reflect reality in order to know that it is an illusion?

    Right. A process of learning, of feedback, of correcting based on hypothesis and test.

    ReplyDelete
  166. The existence of God is not a neutral issue that no one really loses sleep over (like, say, the ontological status of propositions). On the contrary, it's an explosively contentious issue. People either want it to be true or want it to be false. Those who want to believe in God will find a way to believe in Him, and those who do not want to believe in God will find a way to disbelieve in Him, very often going so far as to deny His existence.

    It's as simple as that.

    ReplyDelete
  167. John Thayer Jensen said:

    I think more than one has recommended to Djindra the writings of C. S. Lewis - I have, off-list.

    I remember reading his The Screwtape Letters and thought it quite powerful. I disagreed with the premise of course, but as a metaphor, as psychology, I thought it top-notch.

    It is certainly possible, without absurdity - in fact, without losing much in the way of plausibility - to deny the existence of God. It seems to me, on the other hand, deeply plausible to believe in His existence.

    Maybe. I actually tend somewhat to the idea of ignosticism – “the view that a coherent definition of God must be presented before the question of the existence of god can be meaningfully discussed”. Many different ideas and possible benefits. What I tend to object to are dogmatic assertions about and positions on the topic. I’m reminded of a phrase from Oliver Cromwell: I beseech you, in the bowels of Christ, think it possible that you may be mistaken.

    I think I have good reason to believe - but I have vastly stronger reasons for wanting it to be true.

    Naturally understandable why many would wish it to be true – a notable, if not profound, desire for immortality has manifested itself throughout our history. The trick, I think, is to not let our desires get the better of our intellects. As the American moralist Phillip Wylie averred, “We aspire to the discipline of the instinct by the heart and mind”.

    ReplyDelete
  168. @Steersman:
    a notable, if not profound, desire for immortality has manifested itself throughout our history.

    Indeed :-)

    But, if I may say so, it goes deeper than that. 19th Century man appears to have longed to have his sins forgiven. Perhaps it is the sickness of 20th Century man (I was born in 1942) to long to find meaning in existence. That is surely the father of existentialism. For some, of my generation, at least, and particularly some who did rather go through a scientific sort of training (well, in my case, linguistics, if that is at all scientific :-)), and then experienced Thomas Kuhn - radical scepticism and the fear of the void is the real dread.

    But - again - if I didn't think I had reasons, I would be just another existentialist trying to make his own reality.

    The very first and great reason to read Lewis is that he is such a marvellous writer! I confess I find it difficult to be patient reading poor writing :-)

    ReplyDelete
  169. Hi Ed,

    Thanks for your response. I agree that simply saying that I regard most metaphysics as nonsense does not constitute a substantive criticism of your work. The key clarification I would like to make here is that I view my criticism of metaphysics as empirical rather than metaphysical. That is, we find empirically that metaphysics never leads to anywhere near consensus among experts of good-faith (which I take as a necessary condition for something to count as having been reliably demonstrated).

    First let me return to Dawkins and Coyne: I agree that Dawkins has some 1ish tendencies. I think he does view the argument from design as having had some merit until the discovery of evolution, and in that respect he is a type 2 atheist. It's unclear to me whether he regards his other arguments as ones which should have been accessible to people at all times, or as arguments which only make sense in light of the modern scientific worldview (I view them as the latter type). When Dawkins makes his case against God on the grounds that a creator God must be complex or when he argues against Aquinas, your reading of his argument is that he is doing metaphysics exceptionally badly. My reading of his argument is that he is not engaging at all with the purportedly "simple" God of the theologians, but rather pointing out why the notion of God makes no sense and has no explanatory power within a scientific worldview. Dawkins is simply not talking to theologians here, he is talking to the "median religious believer" who has some respect for science but has never given a second thought to questions of metaphysics. He is engaging with Aquinas only insofar as Aquinas is relevant to this median religious believer (I admit I have not read every word of the God Delusion either so if there is a point at which he says otherwise I am open to being corrected). Now, I think you're right that Dawkins is largely unfamiliar with the kinds of arguments you would make in Aquinas, but this is because he doesn't care what metaphysicians think because he regards the whole discipline as folly and sophistry. I would presume that this is a conclusion he draws in light of modern science which has provided more reliable ways of obtaining knowledge. If this is right, he would argue that Aristotle was not wrong-headed for his time, he was just proven wrong by history, but I don't know for sure that this is Dawkins' view. I'm not sure which of Coyne's arguments cause you to think he is a type 1 atheist rather than a type 2. The arguments of his that I am familiar with all start from premises founded in modern science (e.g. that evolution shows there are not two unique common ancestors to all of humanity). Now, you might argue that he is wrong to think that this or that scientific fact refutes the interpretation you give to the story of Adam and Eve, but it seems to me he is still a type 2 atheist in your sense because he argues that the advances of modern science undermine religious beliefs, not that they have always been completely unreasonable and easily refuted.

    (continued)

    ReplyDelete
  170. (continued)

    I do understand the claim that you are making that there are types of knowledge (or at least true claims) which are not empirical. Like most metaphysical claims, I find this claim difficult to evaluate, I lean towards accepting it but am uncertain (e.g. consider Hilary Putnam's article, "There is at least one a priori truth" and subsequent revisions). You further say that the metaphysical claims you advocate, because they are necessary presumptions for any empirical reasoning to work, "are more well established" than any predictive theory. By analogy, one might argue that something like the law of non-contradiction or modus ponens is more well-established than any mathematical proof which relies on those rules of reasoning. But your argument here is just obviously wrong. Your Aristotelian metaphysics is not more well-established than something like Newtonian mechanics. Many well-intentioned experts disagree about whether your theory is correct, but none disagree about whether Newtonian mechanics is a correct theory of nature at least within the regimes at which it applies (e.g. non-relativistic speeds, non-quantum energy levels...). Now don't misunderstand me: I'm not saying that consensus is what determines truth. I'm saying that one very reliable way we can assess whether something about the world has been adequately demonstrated to be true is by asking whether this demonstration convinces the vast majority of good-faith experts. By this natural standard, Aristotelian metaphysics fails completely. So my question for you is - if Aristotelian metaphysics is so well-established, more established in your view than any scientific theory - why do so many philosophers disagree that it gives a correct foundation for empirical science?

    ReplyDelete
  171. Stone Top,

    "To develop the technology they were using empiricism... unless you can point to an example of technology that was built without it."

    You are confusing empiricism with the empirical. Simply using empirical data does not make one an empiricist; one has to give universal priority to the empirical to be an empiricist. And you can build technology paying attention to the empirical without making any claim that the empirical is the way to get all knowledge worth having.

    ReplyDelete
  172. djindra,

    "If I want to be a stickler, empiricism is not a philosophy, it's a theory of knowledge -- a branch of philosophy."

    That's not being a stickler. That's nitpicking to the point of being evasive. The point was that empiricism is not science but is philosophy, and you were undercutting the usefulness of philosophy, which is what underpins science through empiricism.

    "Yes, "it works" is somewhat like Pragmatism but let's not forget science predates James and Dewey by a a couple of centuries. I don't think there's much doubt that science sits at the pinnacle of empiricism and in this century there's no good reason to talk of empiricism without referring to science."

    Did science use the "it works" justification for itself to the extent that it's used in this century before that? I haven't really seen that, and as you admit that notion is very much Pragmatic. But again you seem to dodge the point that your science depends an awful lot on philosophy in order for it to get to the point where you can say anything useful about it.

    "Science as we know it today is diametrically opposed to rationalism. In no way does it underpin science."

    I didn't actually say rationalism did. I said that science wouldn't be what it is without the debates and issues raised against it by rationalism, and claimed that the hypothetico-deductive system was highly influenced by rationalism. To be specific, it was formed by empiricists noting what reason did really well, and letting that guide the determination of what sense experiences are to be considered to actually reflect reality. Again, naive empiricism simply doesn't work.

    "And I don't know what you mean about science being useless in working out general and abstract concepts. Theories are general and abstract concepts."

    Not in the same sense, but they are more conceptual. But how do you say, work out what a moon should be scientifically, or what an infinite set of numbers is?

    ReplyDelete
  173. The existence of God is not a neutral issue that no one really loses sleep over (like, say, the ontological status of propositions). On the contrary, it's an explosively contentious issue.

    It's also a personal choice (see John Thayer Jensen - "...I could choose to doubt. I do not so choose.") that can't be forced in any meaningful way. Though, in many societies, even today, an outward compliance with the majority norm is advisable for a quiet life. Hence US accommodationists! :)

    People either want it to be true or want it to be false.

    I don't agree with the assertion. I know one ex-Catholic who expresses regret at no longer being convinced by the dogma. In my more expansive moments, I could agree that some aspects of some religious dogma could be appealing if true but, for me, truth outweighs emotional appeal. Actually, the supposition that there is variability in an emotional need for belief is, for me, the bones of an explanation as to why there is a plurality of belief across time and territory and that most of us follow the religion of our cultural and ethnic roots.

    Those who want to believe in God will find a way to believe in Him, and those who do not want to believe in God will find a way to disbelieve in Him, very often going so far as to deny His existence.

    As I have said before, the only civilised society is one that permits freedom of thought. Belief is a personal choice and I grant to everyone the right that I demand for myself.

    I do have to ask, what does it matter to anonymous (of September 16, 2011 10:54 PM) if someone else "denies His existence"? It's not "The Neverending Story" effect is it?

    It's as simple as that.

    ?

    ReplyDelete
  174. No they where using science. Empiricism is a philosophy it is not science.

    So, again, what bridge was built without using "empiricism"?

    ReplyDelete
  175. And you can build technology paying attention to the empirical without making any claim that the empirical is the way to get all knowledge worth having.

    You are when you are developing the technology... unless you can give an example of technology that was developed without using empirical knowledge.

    ReplyDelete
  176. I think I have good reason to believe - but I have vastly stronger reasons for wanting it to be true.

    I'm reminded of what Bertrand Russell said... "Don't let your self be distracted by what you wish to believe, but look only at what are the facts, and what is the truth that the facts bear out."

    ReplyDelete
  177. But again you seem to dodge the point that your science depends an awful lot on philosophy in order for it to get to the point where you can say anything useful about it.

    Rather manifestly not true... If I am falling from an airplane I don't need to work through the philosophy of what is happening before the parachute works.

    ReplyDelete
  178. Those who want to believe in God will find a way to believe in Him, and those who do not want to believe in God will find a way to disbelieve in Him, very often going so far as to deny His existence.

    True, however only one of those two states can be correct... either a described deity exists or it doesn't.

    ReplyDelete
  179. My feserian type argument for undetectable head dancing gremlins:

    One of the gods I believe in is the necessarily existing being Sudoku-weh. Human beings play Sudoku, a game which requires calculation, imagination, and abstraction. It is conceptually impossible for these abilities to have evolved naturally. Therefore, at some time in the past, Sudoku-weh planted undetectable gremlins on the heads of humans which gives them Sudoku playing abilities.

    There is no evidence against this supposition.

    See guys? It's easy to make a feserian-type philosophical argument if you just start making stuff up. Which is exactly what Feser does. He makes stuff up, safeguards it from empirical investigation by calling it "metaphysics", and continues believing in Fairy Tales like Adam & Eve.

    His manufactured story about a god injecting souls into Adam and Eve is not reasonable. It is not something a serious philosopher would do. Rather, it is the move an apologist will make to protect his dogma come what may.

    It is a desperate post hoc dog and pony show.

    ReplyDelete
  180. edit -

    It is a desperate ad hoc dog and pony show.

    ReplyDelete
  181. I have visited here many times (Hi, Ed) and I got sent here, today, via Mark Shea's blog. I don't know if I will have anything useful to say in any comments I might make, since I see no contradictions between science and the Faith, but I am enjoying the discussion in the comment box.

    The Chicken

    ReplyDelete
  182. Oops... That was me.

    The Chicken

    ReplyDelete
  183. BeingItSelf said:

    "One of the gods I believe in is the necessarily existing being Sudoku-weh."

    First. You'll need 8 volumes, 3,000 pages, and 8,000,000 words defining new words and concepts, splitting hairs, and covering your tracks with opaque word salad. Then no layman will really know quite where you begged the question or jumped the shark or circularized yourself.

    Then whenever they corner you. Pick up and move the goalposts. Make an post hoc rationale for what it really means. But remember. No matter what. Hold to the story that there was a 'fall'. No matter what. There has to be.

    ReplyDelete
  184. BeingItSelf,

    Don't forget to talk about 'purpose'. It's important. Make up a purpose for everything. Then, use motivated reasoning to get you to your deity. He has to be necessary for the purpose.

    Maybe he could ever spring forth from the femur of the creator god.

    Here's just a sample of what your diatribe might sound like:

    I answer that, It was right for the woman to be made from a rib of man.

    First, to signify the social union of man and woman, for the woman should neither "use authority over man," and so she was not made from his head; nor was it right for her to be subject to man's contempt as his slave, and so she was not made from his feet.

    Secondly, for the sacramental signification; for from the side of Christ sleeping on the Cross the Sacraments flowed--namely, blood and water--on which the Church was established.

    Reply to Objection 1. Some say that the woman's body was formed by a material increase, without anything being added; in the same way as our Lord multiplied the five loaves. But this is quite impossible. For such an increase of matter would either be by a change of the very substance of the matter itself, or by a change of its dimensions. Not by change of the substance of the matter, both because matter, considered in itself, is quite unchangeable, since it has a potential existence, and has nothing but the nature of a subject, and because quantity and size are extraneous to the essence of matter itself. Wherefore multiplication of matter is quite unintelligible, as long as the matter itself remains the same without anything added to it; unless it receives greater dimensions. This implies rarefaction, which is for the same matter to receive greater dimensions, as the Philosopher says (Phys. iv). To say, therefore, that the same matter is enlarged, without being rarefied, is to combine contradictories --viz. the definition with the absence of the thing defined.

    Wherefore, as no rarefaction is apparent in such multiplication of matter, we must admit an addition of matter: either by creation, or which is more probable, by conversion. Hence Augustine says (Tract. xxiv in Joan.) that "Christ filled five thousand men with five loaves, in the same way as from a few seeds He produces the harvest of corn"--that is, by transformation of the nourishment. Nevertheless, we say that the crowds were fed with five loaves, or that woman was made from the rib, because an addition was made to the already existing matter of the loaves and of the rib.

    Reply to Objection 2. The rib belonged to the integral perfection of Adam, not as an individual, but as the principle of the human race; just as the semen belongs to the perfection of the begetter, and is released by a natural and pleasurable operation. Much more, therefore, was it possible that by the Divine power the body of the woman should be produced from the man's rib.

    From this it is clear how to answer the third objection.

    ReplyDelete
  185. Anon,

    Thanks for the support. I think I can do it in less than 8,000,000 words. Much much less.

    Since I have now given my argument for gremlins, I anticipate that my interlocutors will move the goal post and demand an argument for Sudoku-weh. I've already got one in the can.

    ReplyDelete
  186. The Summa is always fertile ground for finding meaningless gobbledygook, but I have a less elaborate strategy.

    ReplyDelete
  187. Matteo,

    A very interesting recommendation. Thanks.

    ReplyDelete
  188. Anonymous,

    "...and because quantity and size are extraneous to the essence of matter itself." & etc...

    Very funny stuff. I knew there was a reason for sticking with this site.

    ReplyDelete
  189. The argument for Gremlins is nothing more than an argument for or against the existence of another being along side other beings.

    Which for those of us who read TLS has nothing to do with God since He is BeingItself(the real one not the Gnu sophist who posts here) and not a being along side other beings.

    The goal posts are then moved to pull an argument Lewis Wolpert tried unsuccessfully on William Lane Craig except instead of "Gremlins" Wolpert talked about a "Computer" creating the universe.

    http://www.youtube.com/watch?v=sCUE10dY3Rc

    We have heard this crap before can't you Gnus come up with an original argument for once?

    Would it kill ya to read some Quintin Smith or some William Rowe and give us a real argument from real Atheist philosophers instead of these brain dead Pop Atheists like Dawkins or Myers?

    Seriously!

    ReplyDelete
  190. Which for those of us who read TLS has nothing to do with God since He is BeingItself(the real one not the Gnu sophist who posts here) and not a being along side other beings.

    So the existence of invisible, undetectable, Sudoku playing gremlins is consistent with your metaphysics?

    ReplyDelete
  191. >So, again, what bridge was built without using "empiricism"?

    The one built by Young Earth Creationists with Engineering degrees who do not believe Reality is something that can only be known by science alone sans philosophy.

    Now you are confusing and conflating the methods of scientific engineering with philosophy.

    Another category mistake.

    Like my hypothetical Gnu Atheist who denies the existence Andromeda galaxy because he can't see it under his microscope example.

    Arguing the success of the Microscope in detecting micro objects does not vindicate the anti-Galaxy because I can't see it under the microscope philosophy. Nor is it necessary to believe this philosophy to use a microscope. Nor is the use of a microscope a practical manifestation of this philosophy.

    Thus the practice of science or the use of engineering principles to construct technology is not a practical manifestation of Empiricist philosophy. It really has nothing to do with it I'm sorry.

    ReplyDelete
  192. Verbose Stoic,

    "Did science use the 'it works' justification for itself to the extent that it's used in this century before that?

    Absolutely. Read Bacon, for example. It was an exasperation with the Schoolmen and what obviously didn't work, and still doesn't work, that led to science in the first place.

    "The point was that empiricism is not science but is philosophy, and you were undercutting the usefulness of philosophy, which is what underpins science through empiricism."

    My point is that you are nitpicking for purposes of confusion. You seem to complain merely that I use empiricism and science in the same breath without bothering to tediously define the distinctions. In your earlier post you seemed to think I was arguing "that empiricism justifies science." I wouldn't argue that. If anything, I'd argue science justifies empiricism.

    "But again you seem to dodge the point that your science depends an awful lot on philosophy in order for it to get to the point where you can say anything useful about it."

    There is no doubt that science -- derogatorily referred to as scientism -- is seen around here as something separate from philosophy. So when I merely stick with the common usage you and others whine. Make up your minds. If you want science to be understood as philosophy I have no problem with that. It's just language. But stick to it. Don't do -- like Feser does -- whine that the philosophers commonly called scientists don't understand philosophy. OTOH, as long as they are treated as non-philosophers around here I have no problem sticking with the same distinction.

    ReplyDelete
  193. "One of the gods I believe in is the necessarily existing being Sudoku-weh. Human beings play Sudoku, a game which requires calculation, imagination, and abstraction. It is conceptually impossible for these abilities to have evolved naturally. Therefore, at some time in the past, Sudoku-weh planted undetectable gremlins on the heads of humans which gives them Sudoku playing abilities.
    There is no evidence against this supposition.
    See guys? It's easy to make a feserian-type philosophical argument if you just start making stuff up. Which is exactly what Feser does."

    Okay, I now see that you have no idea whatsoever what the terms 'philosophical' and 'argument' (or, indeed, 'logic') even mean, so I see no need to continue this conversation.

    ReplyDelete
  194. Fake Herzog,

    From the link you posted: “My grandfather seemed to me stricken . . . like a man everlastingly struck by lightning." I like that. I picture Cotton Mather. I'll locate a copy. Thanks.

    ReplyDelete
  195. I can't resist...

    BeingItself: "One of the gods I believe in is the necessarily existing being Sudoku-weh. Human beings play Sudoku, a game which requires calculation, imagination, and abstraction. It is conceptually impossible for these abilities to have evolved naturally. Therefore, at some time in the past, Sudoku-weh planted undetectable gremlins on the heads of humans which gives them Sudoku playing abilities."

    Let's break this down as a syllogism:

    Premise 1: Human beings play Sudoku, a game which requires calculation, imagination, and abstraction.

    Undeniably true.

    Premise 2: It is conceptually impossible for these abilities to have evolved naturally.

    Controversial and not undeniably true. (The syllogism fails here, but we'll continue for the fun of it...)

    Conclusion: Therefore, at some time in the past, Sudoku-weh planted undetectable gremlins on the heads of humans which gives them Sudoku playing abilities.

    This does not follow from the premises - even if both premises were true. Your attempt at an argument for the existence of Sudoku-weh fails.

    Now - if you want to see how real philosophical arguments are done - go and use these same methods to analyze the Five Ways.

    ReplyDelete
  196. "Thus the practice of science or the use of engineering principles to construct technology is not a practical manifestation of Empiricist philosophy. It really has nothing to do with it I'm sorry."

    So we have one guy claiming empiricism underpins science, and another who claims "Empiricist philosophy" has nothing to do with science and engineering. (And if I remember correctly the second guy has taken a different position with me.) No wonder so many scientists and engineers ignore this "philosophic" bantering.

    ReplyDelete
  197. Daniel Smith,

    I guess it went right over your head that BeingItself was constructing a more or less accurate parody of what passes for logic in things like Philosophy of Mind and TLS.

    ReplyDelete
  198. edit:

    "Your attempt at an argument for the existence of Sudoku-weh implanted, undetectable gremlins, on the heads of humans, fails."

    Also, I should have said "The argument fails here" instead of "The syllogism fails here"

    ReplyDelete